You are on page 1of 93

Test Paper :5 Paper Type Test Date Posted By : Aptitude - Numerical : 31 December 2010 : Junior G

Test Location : Techno India

#1 A can do a work in 36 days, B can do the work in 72 days and C can do the work in 78 days. The three take turns in working, i.e. A works on the first day, B works on the second day, C on the third day and so on. Which day will they finish the work? 55 th day 19 th day 54 th day 17 th day #2 Consider two tumblers, the first containing one litre of milk and the second containing one litre of coffee. Suppose you take one cup of milk out of the first tumbler and pour it into the second tumbler. After mixing you take one cup of the mixture from the second tumbler and pour it back into the first tumbler. Which one of the following statements holds now? There is less coffee in the first tumbler than milk in the second tumbler. There is more coffee in the first tumbler than milk in the second tumbler. There is as much coffee in the first tumbler as there is milk in the second tumbler. None of the statements holds true. #3 In a certain coding system used by the military intelligence, URMILA is written as SACHIN. KANIKA is written as NOROIN and TANISHA is written as INTERNL. Which of the following cannot be the code for NIKETA? TINORK EINORD MLIONR RLIONS #4 Susan made a block with small cubes of 7 cubic cm volume to make a block 4 small cubes long, 4 small cubes wide and 10 small cubes deep. She realises that she has used more small cubes than she really needed. She realised that she could have glued a fewer number of cubes together to look like a block with same dimensions, if it were made hollow. What is the minimum number of cubes that she needs to make the block? 160 32 128 79 #5 A certain street contains 100 buildings. They are numbered from 1 to 100. How many 9s are used in numbering these buildings? 21 20 19 9 #6 A power unit is by the bank of a river 1500m wide. It is required to construct a power plant on the side of the river opposite to the power unit and 3000mfrom the unit. A cable has to be laid from the power unit to the power plant. The cost of laying the cable under water and on the land is Rs 100 per m and Rs 10 per m respectively. If we want to minimize the total cost of laying the cable, where should the plant be located? 750 m from the river bank 1500 m from the river bank 1200 m from the river bank 3000 m from the river bank #7 A man speaks truth 3 out of 4 times. He throws a die and reports it to be a 6. What is the probability of it being a 6? 1 3 5 1 /2 /4 /8 /8

#8 The fundamental theorem of arithmetic states that every natural number greater than 1 can be written as a unique product of prime numbers. Now what is the smallest number by which 3380 must be divided in order to make it into a perfect square? 5 8 6 4 #9 Rearrange the following letters to make a word and choose the category in which it fits. RAPETEKA City Fruit Vegetable Bird # 10 How many 8 digit numbers formed using 1,2,3,4,5 with repetition are divisible by 4? 97657 97656 78125 31250

# 11 Find the missing number 8 , 1 , 17 , 4 , 35 , 10 , 71 , ?, 143 22 20 12 21

# 12 The cost of making the robot is divided between materials, labour and overheads in the ratio of 4:5:2. If the materials cost $116, the cost of the robot is 255.2 638 319 261

# 13 Ferrari S.p.A. is an Italian sports car manufacturer based in Maranello, Italy. Founded by Enzo Ferrari in 1928 as Scuderia Ferrari, the company sponsored drivers and manufactured race cars before moving into production of street-legal vehicles in 1947 as Ferrari S.p.A.. Throughout its history, the company has been noted for its continued participation in racing, especially in Formula One, where it has enjoyed great success. Rohit once bought a Ferrari. It could go 2 times as fast as Mohit's old Mercedes. If the speed of Mohit's Mercedes is 32 km/hr and the distance travelled by the Ferrari is 952 km, find the total time taken for Rohit to drive that distance. 29.75 476 15.88 14.88

# 14 A woman has 136 yards of cloth on a single roll, and she wants to divide it into 136 lengths of 1 yard each. It takes her 5 seconds to cut each length. Working non-stop, how long (in seconds) will it take her to cut all 136 pieces? 679 675 680 681

# 15 4 years ago, Aditya's age was twice Anvesh's age. 4 years hence, Aditya's age will be 4/3 times the age of Anvesh. Find Aditya's present age. 5 years 12 years 9 years 4 years

all d questions were vry hard...better u guys solve these hii... my name is prakash ranjan.Im from Kalyani Govt. Engg. College.. TCS come in my college on 7th jan 2011. i want share some of my experience with all of u.... At first we have a aptitude test of 80 min.Its online.I just give u some tricks of some problems asked in aptitude... 1. There are two water tanks A and B, A is much smaller than B. While water fills at the rate of 1 liter every hour in A, it gets filled up like, 10, 20, 40, 80, 160 in tank B. (At the end of first hour, B has 10 liters, second hour it has 20 liters and so on). If tank B is 1/32 filled of the 21 hours, what is total duration of hours required to fill it completely? ans.-here part filled is 1/32 then 1/32=1/(2)^5. so ans is 21+5. 2. 6 persons standing in queue with different age group, after two years their average age will be 43 and seventh person joined with them. Hence the current average age has become 45. Find the age of seventh person? ans.-Total age of 6 persons is x hours,after two years total age of 6 persons is x+12 Average age of 6 persons is after two years is 43 So (x+12)/6=43,then x=246 After 7th person is added then (246+7th person age)/7=45 7th person age = 69 3. One statement problem-in this problem if atleast all statement are true - first part true and last part false exactly all statement are true - second part true rest is false atmost all statement are true - all are false. 4. The hare starts after the tortoise has covered 1/5 of its distance and that too leisurely3. A hare and a tortoise have a race along a circle of 100 yards diameter. The tortoise goes in one direction and the. The hare and tortoise meet when the hare has covered only 1/8 of the distance. By what factor should the hare increase its speed so as to tie the race? in this problem atfirst find the ratio of the distance left to cover the turtle and distance by the hare. means-(87.5-20)/12.5=5.4 and then 87.5/12.5=7 and multiply these two 5.4*7=37.8 5. alok and Bhanu play the following min-max game. Given the expression N=9+X+Y-Z Where X, Y and Z are variables representing single digits (0 to 9), Alok would like to maximize N while Bhanu would like to minimize it. Towards this end, Alok chooses a single digit number and Bhanu substitutes this for a variable of her choice (X, Y or Z). Alok then chooses the next value and Bhanu, the variable to substitute the value. Finally Alok proposes the value for the remaining variable. Assuming both play to their optimal strategies, the value of N at the end of the game would be. ans.- in this problem there is 3 patern 1st X+Y-Z=11 X*Y-Z=18 X-Y-Z=2 u just add the to the no given. 6.- For the FIFA world cup, Paul the octopus has been predicting the winner of each match with amazing success. It is rumored that in a match between 2 teams A and B, Paul picks A with the same probability as A's chances of winning. Let's assume such rumors to be true and that in a match between Ghana and Bolivia, Ghana the stronger team has a probability of 2/3 of winning the game. What is the probability that Paul will correctly pick the winner of the Ghana-Bolivia game? ans.- hare the answer is - 2/3*2/3+1/3*1/3=5/9 7.-After the typist writes 12 letters and addresses 12 envelopes, she inserts the letters randomly into the envelopes (1 letter per envelope). What is the probability that exactly 1 letter is inserted in an improper envelope? ans- 0 8.- There are two boxes, one containing 10 red balls and the other containing 10 green balls. You are allowed to move the balls between the boxes so that when you choose a box at random and a ball at random from the chosen box, the probability of getting a red ball is maximized. This maximum probability is ans.- the tricks is - 1/2*1+(n-1)/(n-1+m)*1/2 here n is the no. which have to be max. and m is the another no. here m is green ball and n is red ball.here ans is 14/19 9.- A and B play a game of dice between them. The dice consist of colors on their faces (instead of numbers). When the dice are thrown, A wins if both show the same color; otherwise B wins. One die has 4 red face and 2 blue faces. How many red and blue faces should the other die have if the

both players have the same chances of winning? ans.- here the ans is 3red and 3 blue. 10.- On planet zorba, a solar blast has melted the ice caps on its equator. 8 years after the ice melts, tiny plantoids called echina start growing on the rocks. echina grows in the form of a circle and the relationship between the diameter of this circle and the age of echina is given by the formula d = 4 * sqrt (t 8)for t = 8 Where the represents the diameter in mm and t the number of years since the solar blast. Jagan recorded the time of some echina at a particular spot is 24 years then what is diameter? ans- this is the simple problem just put the value of t and find d. 11.- A circular dartboard of radius 1 foot is at a distance of 20 feet from you. You throw a dart at it and it hits the dartboard at some point Q in the circle. What is the probability that Q is closer to the center of the circle than the periphery? ans.- here the tricks is (n/2)^2/n^2 here n is the radius of circle. 12.- 36 people {a1, a2, ..., a36} meet and shake hands in a circular fashion. In other words, there are totally 36 handshakes involving the pairs, {a1, a2}, {a2, a3}, ..., {a35, a36}, {a36, a1}. Then size of the smallest set of people such that the rest have shaken hands with at least one person in the set is. ans.- hare the tricks is if the handshake is circular and given no is even-then n/2 if odd then n/3. and if hand shake is non circular then n-1. 13.- Alice and Bob play the following coins-on-a-stack game. 20 coins are stacked one above the other. One of them is a special (gold) coin and the rest are ordinary coins. The goal is to bring the gold coin to the top by repeatedly moving the topmost coin to another position in the stack. Alice starts and the players take turns. A turn consists of moving the coin on the top to a position i below the top coin (0 = i = 20). We will call this an i-move (thus a 0-move implies doing nothing). The proviso is that an i-move cannot be repeated; for example once a player makes a 2-move, on subsequent turns neither player can make a 2-move. If the gold coin happens to be on top when it's a player's turn then the player wins the game. Initially, the gold coinis the third coin from the top. ans.- Ans is in order to win, Alice's first move should be a 1-move.(This is the final ans.) 14.- The citizens of planet nigiet are 8 fingered and have thus developed their decimal system in base 8. A certain street in nigiet contains 1000 (in base 8) buildings numbered 1 to 1000. How many 3s are used in numbering these buildings? ans.- here the tricks is 8*8+8*8+8*8=192 (here the base is 8)if the base is n then n*n+n*n+n*n. 15.- Six friends decide to share a big cake. Since all of them like the cake, they begin quarreling who gets to first cut and have a piece of the cake. One friend suggests that they have a blindfold friend choose from well shuffled set of cards numbered one to six. You check and find that this method works as it should simulating a fair throw of a die. You check by performing multiple simultaneous trials of picking the cards blindfold and throwing a die. You note that the number shown by the method of picking up a card and throwing a real world die, sums to a number between 2 and 12. Which total would be likely to appear more often 8,9 or 10? a) 8 b) All are equally likely c) 9 d) 10 ans.- here the ans is 8. because 8 comes in 3 types(4+4,5+3,6+2),9 comes in 2 types(5+4,6+3),and 10 comes 2 types(5+5,6+4). 16.- How many of 14 digit numbers we can make with 1,2,3,4,5 that are divisible by 4. Repetitions allowed. ans.- here ans is 5^(n-1) n is the 14. 17.- Which is the smallest no which divides 2880 and gives a perfect square? a) 4 b) 9 c) 3 d) 5 ans.- 2880/5=576 and 576=24*24. 18.- 3 persons a,b,c were there A always says truth,B lies on Monday,tusday,& Wednesday.but C lies on thrusday,Friday & saturday .one day A saidthat B & C said to A that B said yesterday way one of the days when I lies,C said thatyesterday way one of the days when I lies too.then which day was that? ans.- the ans of all this type is Thursday. 19.-There 10 programers, type 10 lines with in 10 minutes then 60lines can type within 60 minutes. How many programmers are needed?

ans.- we have 3 question of this type the solution is based on time and work. 20.-amrith told to Anand in front of a Photo that He is the son of my fathers son.Find who is in the picture if amrith have no brothers and sisters. ans.- the ans is him means amrith itself. 21.- In T.Nagar the building were numbered from 1 to 100.Then how many 4s will be present in the numbers? ans.- Here if the n's is given then ans is 20. n's is from 2 to 9. and if 1 is given then ans is 21. 22.- If a and b are mixed in 3:5 ration and b and c are mixed in 8:5 ration if the final mixture is 35 liters, find the amount of b? ans.- b/(a+b+c)*35. 23.- If there are 30 cans out of them one is poisoned if a person tastes very little he will die within 14 hours so if there are mice to test and 24 hours to test, how many mices are required to find the poisoned can? ans.- 6 24.- A man whose age is 45 yrs has 3 sons named John, Jill, jack. He went to a park weekly twice. He loves his sons very much. On a certain day he found the shop keepers selling different things. An apple cost 1penny, 2chocalate costs 1penny & 3 bananas cost 1 penny. He has bought equal number of apple, chocolate & banana for each son. If the total amount he invest is 7 penny then how many he has bought from each piece for his son? ans.- 1 apple,2 chocklets,1bananas.simply 1 2 1. 25.- There are 1000 pillars for a temple. 3 friends Linda, Chelsey, Juli visited that temple. (Some unrelated stuff) Linda is taller than Chelsea and taller than 2 of 1000 pillars. Julia is shorter than Linda. Find the correct sentence? ans.-here p>j 26.-A lady has fine gloves and hats in her closet- 18 blue, 32 red, and 25 yellow. The lights are out and it is totally dark. In spite of the darkness, she can make out the difference between a hat and a glove. She takes out an item out of the closet only if she is sure that if it is a glove. How many gloves must she take out to make sure she has a pair of each color? ans.- Here the tricks is at first take the ball is max. and then take second max. no. and 2. here ans is 32+25+2. 27.- A scientist was researching on animal behavior in his lab. He was very interested in analyzing the behavior of bear. For some reason he travelled 1mile in north direction & reached at North Pole. There he saw a bear. He then followed the bear around 1 hr with a speed of 2km/hr in east direction. After that he travelled in south direction & reached at his lab in2 hrs. Then what is the color of the bear? ans.- ans is white bear. Thats all I have attend 34 questions and I just cleared my aptitude test. now my interview is on 7th jan. I have my interview of about 30 min. he just asked every thing from me. don't get tensed just give all the answer confidently. my HR and technical question is 1.- Why TCS? 2.- Whats ur week point and strength. 3.- What the question u got from the students. 4.- Draw the circuit diagram of induction motor. 5.- What is short term and long term sheduling. 6.- What is the diff. between malti tasking and multithreading. 7.- Who is the owner of microsoft and linux. 8.- Proof the equivalant resistance of parellel circuit. 9.- Name the os which have u used in ur life. 10.-What is pointer and panildrom.write program.

Quantitative aptitude test (35 questions, 80 minutes) 1. A sheet of paper has statements numbered from 1 to 20. for each value of n from 1 to 20, statement says 'At least n of the statements on this sheet are true.' which statements are true and which are false? 2. Alok and bhanu play the following min-max game. Given the expression N=46+X+Y-Z, where X,Y and Z are variables representing single digits (0 to 9) alok woul like to maximize N while bhanu would like to minimize it. Towards this end, alok chooses a single digit number and bhanu substitues this for a variable of her choice (X, Y or Z) Alok then chooses the next value and bhanu the variab le to substitute the value. Finallyu aloka proposes the value for the remaining variable. Assuming both play to their optimal strategies the value of

N at the end of the game would be (a) 37 (b) 57 (c) 64 (d) 55 3. The pacelength P is the distance between the rear of two consecutive footprints for men the formula n/P=110 gives an approximate relationship between n and P where n= number os steps per minute and P= pacelength in meters. Bernard knows his pacelength is 97cm the formula applies to Bernards walking. Calculate Bernards walking speed in kmph. (a) 106.70 (b) 8.82 (c) 6.21 (d) 11.34 4. Anoop managed to draw 7 circles of equal radii with their centres on the diagonal of a square such that the two extreme circles touch two sides of the square and each middle circle touches two circles on either side. Find the ratio of the side of the square to radius to the circles. You may assume that squate toot of 2 is 1.4 (a) 13.90 :1 (b) 10.40 :1 (c) 11.80 :1 (d) 15.90 :1 5. Alok is attending a workshop how to do more with less and todays theme is working with fewer digits. The speakers discuss how a lot of miraculous mathematics can be achived if mankind (as well as womankind) had only worked with fewer digits. The problem posed at lthe end of the workshop is how many 6 digit numbers can be formed using the digits 1, 2,3,4,5 (but with repetition) that are divible by 4? Can you help alok find the answer? (a) 3125 (b) 3906 (c) 1250 (d) 3907 6. Given 3 lines in the plane such that the points of intersection from a triangle with sides of length 20, 20 and 11 the number of points equidistant from all the 3 lines is (a) 0 (b) 3 (c) 1 (d) 4 7. After the typist writes 100 letters and addresses 100 envelopes she inserts the letters randomly in to the envelopes(1 letter per envelope). What is the probability that exactly 1 letter is inserted in an improper envelop? (a) 0 (b) 1-1/100 (c) 1/100! (d) 1/100 8. The teacher is testing a students proficiency in arithmetic and poses the following question: 1/3 of a number is 3 more than 1/6 of the same number. What is the number? Can you help the student find the answer? (a) 19 (b) 6 (c) 17 (d) 18 9. The IT giant tirnop has recently crossed a head count of 150000 and earnings of $7billion. As one of the forerunners in the technology front, Tirnop continues to lead the way in products and services in India.At tirnop, all programmers are equal in every respect. They receive identical salaries and also write code at the same rate. Suppose 13 such programmers take 13 minutes to write 13 lines of code in total. How many lines of code can be written by 104 programmers in 104 minutes: ( a) 104 (b) 832 (c) 13 (d) 8 10. A sheet of paper has statements numbered from 1 to 16 for all values of n from 1 to 16 statement n says: Exactly n of the statements on this sheet are false. Which statements arte true and which are false? (a) The second last statement is true and the rest are false. (b) The even numbered statements are true and the odd numbered statements are false. (c) All the statements are false. (d) The odd numbered statements are true and the even numbered statement are false. 11. 45 suspects are rounde by the police and questioned about a bank robbery. Only one of them is guilty. The suspects are made to stand in a line and each person declares that the person next to him on his right is guilty. The rightmost person is not questioned. Shich of the following possibilities are ture? A. all the suspectrs are lying. B. the leftmost suspect is

guilty. C. the rightmost suspect is guilty. (a)A and C (b)A and B (c)B only (d)A only 12. 45 people {a1, a2, a45} meet and shake hands in a circular fashion. In other words, there are totally 45 handshakes involving the pairs, {a1, a2}, {a2,a3},..{a44,a45},{a45,a1}. Then the size of the smallest set of people such that the rest have shaken hands with at least one person in the set is (a)16 (b)22 (c)15 (d)14 13.The citizzens of planet oz are 8 fingered and thus habve developed a number system in base 8. A certain street in Oz contains 1000 buildings nubered from 1 to 1000. How many 4s are used in numbering these buildings ? Express your answer in base 10. (a)64 (b)96 (c)192 (d)256 14. Middle earth is fictional land inhabited by hobbits elves dwarves and men. The hobbits and the elves are peaceful creatures who prefer slow silent lives and appreciate nature and art. The dwarves and the men engage in physical games. The game is as follows. A tournoi is one where out of the two teams that play a match the one that loses get eliminated the matches are played in different rounds where in every round; half of the teams get eliminated from the tournament. If there are 6 rounds played in a knock out tournoi how many matches were played? (a) 42 (b) 64 (c) 63 (d) 65 15. Ferrari S. p.A is an italian sports car manufacturer based in Maranello, Italy. Founded by Enzo Ferrari in 1928 as Scuderia Ferrari, the compay sponsored drivers and manufactured race cars bvefore moving into production of street-legal vehicles in 1947 as Ferrari S.p.A through out is history the compay has been noted for its continued participation in racing, especially in Formula One, where it has enjoyed great success. Rohit once bought a Ferrari. It could go 4 times as fast as Mohits old Mercedes. If the speed of Mohits Mercedes is 32 km/hr and the sistance traveelled by the Ferrari is 991 km, find the total time taken for Rohit to drive that distance (a) 7.74 Hours (b) 247 Hours (c) 8 Hours (d) 30 Hours 16. A circula dashboard of radius 1.0 foot is at a distance of 20 feet from you. You throw a dart at it and it hits the dartboard at some point Q I th circle. What is the probability that Q is closer to the center of circle than the periphery? (a) .25 (b) .75 (c) .50 (d) 1.00 17. A hare and a tortoise have a race along a circle of 100 yards diameter. The tortoise goes in one direction and the hare I the other. The hare starts after the tortoise had coveredc 1/7 of its distance and that tooo leisurely. The hare and tortoise meet when the hare has covered only 1/8 of the distance. By what factor shoul the hare increase its speed so as to ties the reace? (a) 8 (b) 41.00 (c) 56 (d) 7 18. On the planet Oz there are 8 days in a week Sunday to Saturday and another day called Oz day. There are 36 hours in a day and each hour has 90 minutes while each minute has 60 seconds. As on earth the hour hand covers the dial twice every day. Find the approximate angle between the hands of a clock on Oz when the time is 15:40 am (a) 186 degrees (b) 311 degrees (c) 131 degrees (d) 149 degrees 19. For the kings revelry 254 barrels of beer have b een ordered . howerver, it was found that one of them is poisoned. The poison takes effect even if consumed in the tiniest amount after 14 hours. Yhou need to find within 24 hours the poisoned barrel and have at your disposal some beer guzzling mice. The smallest number of mice required to find the poisoned barrel is (a) 9 (b) 8 (c) 254 (d) 7

20. Planet Fourfe resides in 4-dimensional space and thus the currency used by its residents are 3- dimensional objects. The rupee notes are cubical in shape shile their coins are spherical. However the coin minting machinery lays out some stipulations on the size of the coins. A. The diameter of the coins should be at least 16 mm and not exceed 64mm. B. given a coin the diameter of the next larger coin is at least 50% greater. C. the diameter of the coin must always be na integer. You are asked to design a set of coins of different diameters with thers requirements and your goal is to desigh as many coins as possible. How many coins can you design? (a) 6 (b) 2 (c) 4 (d) 3 21. There are two boxes, one containing 24 red balls and the other containing 38 green balls. You are allowed to move the balls between the boxes so that when you choose a box at random and a ball at random from the chosen box the probability of getting a red ball is maximized. This maximum probability is (a) .50 (b) .39 (c) .69 (d) .19 22. Given a collection of points P in the plane a 1-set is a point in P that can be separated from the rest by a line; i.e the pint lies on one side of the line while the others lie on the other side. The number of 1-sets of P is denoted by n1(P) . The maximum value of n1(P) over all configurations P of 4 points in the plane is (a) 3 (b) 2 (c) 4 (d) 5 23. There are two water tanks A and B , A is much smaller than B. While water fills at the rate of one litre every hour in A, it gets filled up like 10,20,40,80,160. in tank B. (At the end of first hour, B has 10 litres, second hour it has 20, and so on). If tank B is 1/8 filled after 23 hours, what is the total duration required to fill it completely? (a) 27 hours (b) 25 hours (c) 26 hours (d) 3 hours 24. A man has some socks in his drawere 14 identical blue, 20 identical red, and 28 identical black. The lights are out and it is totally dark. How many socks must he take out to make sure he has a pair of each colour? (a) 34 (b) 50 (c) 30 (d) 28 25. As a result of global warming is that the ice of some glaciers is melting. 13 years after the ice disappears, tiny plants, called lichens, start to grow on the rocks. Each lichen grows approximately in the shape of a circle. The relationship between the diameter of this circle and the age of the lichen can be approximated with the formula: d=18*(t-13)for t>13, where d represents the diameter of the lichen in millimeters, and t represents the number of years after the ice has disappeared. Using the above formula calculate the diameter of the lichen , 39 years after the ice has disappeared. (a) 702 (b) 468 (c) 13 (d) 689 26. 6 persons standing in queue with different age group, after two years their average age will be 43 and seventh person joined with them. hence the current average age has become 45. find the age of seventh person? (a) 49 (b) 45 (c) 69 (d) 20 27. There are 5 materials to make a perfume: Lilac, Balsamic, Lemon, and Woody and MI mosaic. To make a perfume that is in demand the following conditions are to be followed: Lilac and Balsamic go together. Woody and MI mosaic go together; Woody and Balsamic never go together. Lemon can be added with any material. All of the following combinations are possible to make a perfume except (a) Balsamic and Lilac (b) Woody and Lemon (c) MI mosaic and Woody (d) MI mosaic and Lilac

28. A lady has fine gloves and hats in her closet- 18 blue- 32 red, 10 white, 25 yellow, 55 purple, 30 orange. The lights are out and it is totally dark inspite of the darkness. She can make out the difference between a hat and a glove. She takes out an item out of the closet only if she is sure that if it is a glove. How many gloves must she take out to make sure she has a pair of each colour of blue, red, yellow? (a) 59 (b) 8 (c) 50 (d) 42 29. Alok and Bhanu play the following min-max game. Given the expression N = 15 + X*(Y Z) Where X, Y and Z are variables representing single digits (0 to 9), Alok would like to maximize N while Bhanu would like to minimize it. Towards this end, Alok chooses a single digit number and Bhanu substitutes this for a variable of her choice (X, Y or Z). Alok then chooses the next value and Bhanu, the variable to substitute the value. Finally Alok proposes the value for the remaining variable. Assuming both play to their optimal strategies, the value of N at the end of the game would be (a) 28 (b) 30 (c) 33 (d) 35 30. A and B play a game between them. The dice consist of colors on their faces(instead of number). When the dice are thrown, A wins if both show the same color, otherwise B wins. One die has 3 red faces and 3 blue faces. How many red and blue faces should the other die have if the both players have if the both players have the same chances of winning? (a) 5 red and 1 blue faces (b) 1 red and 5 blue faces (c) 3 red and 3 blue faces (d) 4 red and 2 blue faces 31. Entry ticket to an exhibition ranges from 1p to 31p. You need to provide exact change at the counter. You have 31p coin. In how many parts will you divide 31p so that you will provide the exact change required and carry as less coins as possible? (a) 21 (b) 31 (c) 6 (d) 32 32. A sheet of paper has statements numbered from 1 to 45. For all values of n from 1 to 45, statement n says At most n of the statements on this sheet are false. Which statements are true and which are false? (a) The odd numbered statements are true and the even numbered are false. (b) The even numbered statements are true and the odd numbered are false. (c) All statements are true. (d) All statements are false. 32. In planet OZ planet there are 8 days, sunday to saturday and 8th day is OZ day. There is 36 hours in a day. What is angle between 12.40? (a) 80 (b) 81 (c) 87 (d) 89 33. For the FIFA world cup, Paul the octopus has been predicting the winner of each match with amazing success. It is rumored that in a match between 2 teams A and B, Paul picks A with the same probability as As chances of winning. Lets assume such rumors to be true and that in a match between Ghana and Bolivia, Ghana the stronger team has a probability of 2/3 of winning the game. What is the probability that Paul will correctly pick the winner of the Ghana-Bolivia game? (a) 5/9 (b) 1/9 (c) 2/3 (d) 4/9 34. The IT giant Tirnop has recently crossed a head count of 150000 and earnings of $7 billion. As one of the forerunners in the technology front, Tirnop continues to lead the way in products and services in India. At Tirnop, all programmers are equal in every respect. They receive identical salaries ans also write code at the same rate.Suppose 10 such programmers take 10 minutes to write 10 lines of code in total. How many lines will be written by 50 programmers in 50 minutes in total? (a) 50 (b) 18 (c) 10 (d) 250

35. Alice and Bob play the following coins-on-a-stack game. 20 coins are stacked one above the other. One of them is a special (gold) coin and the rest are ordinary coins. The goal is to bring the gold coin to the top by repeatedly moving the topmost coin to another position in the stack. Alice starts and the players take turns. A turn consists of moving the coin on the top to a position i below the top coin (0 = i = 20). We will call this an i-move (thus a 0move implies doing nothing). The proviso is that an i-move cannot be repeated; for example once a player makes a 2-move, on subsequent turns neither player can make a 2-move. If the gold coin happens to be on top when it's a player's turn then the player wins the game. Initially, the gold coinis the third coin from the top. Then (a) In order to win, Alice's first move should be a 1-move (b) In order to win, Alice's first move should be a 0-move (c) In order to win, Alice's first move can be a 0-move or a 1-move (d) Alice has no winning strategy I attempted 29 out of 35 and results were declared in about an hour or so and i cleared the test. In our college around 330 appeared for the apti out of which 270 cleared the written test. Interview was held on 30th December 2010, at Techno India Salt LAKE campus. The reporting time for us was at 2.30 p.m. When my turn for in interview came it was 5.40 p.m. My interview went on for around 20 minutes. My name was called by some TCS official and I went inside along with him inside the room. The room where I was interviewed there was 3 interviewer simultaneously interviewing my friends as that slot was especially for our college only. So I went near to him. To be honest i was nervous and my heart was pounding. I said good evening Sir. Then i sat on the chair. Then he was looking throughly through the TCS form which is to be filled and submitted along with my CV. After couple of minutes he asked me Tell me about yourself? I mentioned my name, native place, education and hobbies. As I mentioned my hobbies as internet surfing and chating. He asked what I luck about chating? Why I spent time in chating rather than reading the books? I'm very passionate about chating so I replayed to him in a manner I guess he liked it. Then he went over to my project- my project title "jChat-Chat Client in Java". He asked all the details about the project then he carried on asked to draw the architecture of the project and database part too. Basically J stands for Java. It is basically intranet chat application which is being developed by me as a part of my final year project. Then he asked me about the keys in DBMS: Primary key its features, foreign key. Constraints different types of constraints. Well in my subject of interest I had mentioned operating system and networking but no questions from this two subjects. After that he said we will get bak to you then said said thank you sir have a nice day and left the room. Anyways I would like to thank God and all those who helped me in paving the path for me. Remember 2 thing in apti dont guess and in interview be relaxed. Another thing your are not out unless and until the result is out even if your out here don't wry as because TCS is not the last company so have faith in God. If anything you wish 2 know just mail me or give me call. If I can I will surely help you. Last but not the least I would like to thank Freshersworld.com for their effort. Its an amazing site for a freshers. He can avail the info from here.

APTITUDE TEST: Enlish: 32 Q,20M 10 -synnonym 10-antonym I got anly 4 common . For antonym U prepare from all Model test parer of barron 12 . Don't wast time for preparing synnonym becoz there are lots of synnonym so u can,t

collct all synnonym. 6-fill in the blank .Exatly this fill in the blank came in my paper. 2. FILL THE BLANKS: Observe the dilemma of the fungus; it is a plant, but possesses no chlorophyll. While all other plants put the suns energy to work for them combining the nutrients of ground and air into the body structure._____1______.Chlorophyll is found in these other plants which, having received their energy free from the sun, relinquish it__2__.

In this search of energy, the fungus has become the earths major source of rot and decay. Wherever you see mould forming on a piece of bread, or a pile of leaves turning to compost, or a bloom down tree becoming pulp on the ground, _____3___. With fungus action, the earth would be pooled high with_____4______. In fact, certain plants which contain resins that are ____5_______. Specimen of the redwood tree, for instance can still be found ______6________. A. You are watching a fungus acting B. Resting on the forest floor centuries after having been cut down C. Responsible for decomposition of much plant life D. Fungus must look elsewhere for an energy supply E. Cannot produce their own store of energy F. The dead plant life of past centuries G. At some point in their cycle either to animal or to fungi H. Fungus is vastly different from other plants I. Toxic to fungi will last indefinitely.
One large passage-6marks. I answer all question from passage in guess becos i have no time to read whole passage care fully. QUANTITATIVE: 38 Q-40 M ALL CAME FROM PREVIOUS YR QUESTION. U SHOULD KEEP ON UR MEMORY THE ANSWER OF EACH QUESTION . THERE IS 3 Q FROM VENN DIAGRAM & 3 FROM PIE CHART. 1. Two pencils costs 8 cents, then 5 pencils cost how much Sol: 2 pencils 8 cents => 1 pencil 4 cents Therefore 5 pencils cost = 5 * 4 = 20 cents

2. A work is done by two people in 24 min. one of them can do this work a lonely in 40 min. how much time required to do the same work for the second person. Sol: (A+B) can do the work in = 1/24 min. A alone can do the same work in = 1/40 min. B alone can do the same work in = (A+B)s As = 1/24 1/40 = 1/60 Therefore, b can do the same work in = 60 min 3.A car is filled with four and half gallons of oil for full round trip. Fuel is taken 1/4 gallon more in going than coming. What is the fuel consumed in coming up? Sol Before the trip, car is filled with = 4 gallon of oil Let X be the quantity of fuel consumed for the trip in one direction The fuel consumed while going = X + The fuel consumed while coming = X

Therefore, the fuel consumed for the trip = (X + ) + X = 4 2X + = 4 => 2X = 4 - => 2X = 4 => X= 2. approx Therefore the fuel consumed while coming = 2 gallon 4. Low temperature at the night in a city is 1/3 more than 1/2 high as higher temperature in a day. Sum of the low temperature and highest temp. is 100 degrees. Then what is the low temp? Sol: 40 deg. 5. A person, who decided to go to weekend trip should not exceed 8 hours driving in a day. Average speed of forward journey is 40 m/h. Due to traffic in Sundays, the return journey average speed is 30 m/h. How far he can select a picnic spot? a) 120 miles b) Between 120 and 140 miles c) 160 miles Answer: 120 miles 6. A person was fined for exceeding the speed limit by 10mph. Another person was also fined for exceeding the same speed limit by twice the same. If the second person was traveling at a speed of 35 mph, find the speed limit. Sol: Let x be the speed limit Person A was fined for exceeding the speed limit by = 10mph Person B was fined for exceeding the speed limit by = twice of A = 2*10mph=20mph given that the second person was traveling at the speed of 35mph => 35mph 20mph = 15mph Therefore the speed limit is =15 mph 7.A bus started from bus stand at 8.00am, and after 30 minutes staying at destination, it returned back to the bus stand. The destination is 27 miles from the bus stand. The speed of the bus is 18mph. In return journey bus travels with 50% fast speed. At what time it returns to the bus stand? Sol: 11.00am 8.In a mixture, R is 2 parts S is 1 part. In order to make S to 25% of the mixture, how much r is to be added? Sol: One Part 9. Wind flows 160 miles in 330 min, for 80 miles how much time required. Sol:

10. With 4/5 full tank vehicle travels 12 miles, with 1/3 full tank how much distance travels Sol: ( 5 miles ) 11. A storm will move with a velocity of towards the center in hours, at the same rate how much far will it move in hrs. Sol: ( but the answer is 8/3 or 2 2/3 ) 12. In a two-dimensional array, X (9, 7), with each element occupying 4 bytes of memory, with the address of the first element X (1, 1) is 3000; find the address of X (8, 5). Ans: 3212 13. In the word ORGANISATIONAL, if the first and second, third and forth, forth and fifth, fifth and sixth words are interchanged up to the last letter, what would be the tenth letter from right? Ans:I 14. What is the largest prime number that can be stored in an 8-bit memory? Ans : 251 15. Select the odd one out..a. Java b. Lisp c. Smalltalk d. Eiffel. 16. Select the odd one out a. SMTP b. WAP c. SAP d. ARP 17. Select the odd one out a. Oracle b. Linux c. Ingress d. DB2 18. Select the odd one out a. WAP b. HTTP c. BAAN d. ARP 19. Select the odd one out a. LINUX b. UNIX c. SOLARIS d. SQL SEVER 20. Select the odd one out a. SQL b. DB2 c. SYBASE d. HTTP 21. The size of a program is N. And the memory occupied by the program is given by M = square root of 100N. If the size of the program is increased by 1% then how much memory now occupied? Ans: 0.5%(SQRT 101N) 22. A man, a woman, and a child can do a piece of work in 6 days. Man only can do it in 24 days. Woman can do it in 16 days and in how many days child can do the same work? Ans: 16

23. In which of the system, decimal number 184 is equal to 1234? Ans: 5 24. Find the value of the 678 to the base-7. Ans: 1656 25. Number of faces, vertices and edges of a cube Ans: 6 8 12 26. Complete the series 2, 7, 24, 77,__ Ans: 238 27. Find the value of @@+25-++@16, where @ denotes "square" and + denotes "square root". Ans: 621 28. Find the result of the following expression if, M denotes modulus operation, R denotes round-off, T denotes truncation: M(373,5)+R(3.4)+T(7.7)+R(5.8) Ans:19 29. If TAFJHH is coded as RBEKGI then RBDJK can be coded as? Ans: qcckj 30. G(0)= -1, G(1)=1, G(N)=G(N-1) - G(N-2), G(5)= ? Ans: - 2 31. What is the max possible 3 digit prime number? Ans: 997 32. A power unit is there by the bank of the river of 750 meters width. A cable is made from power unit to power plant opposite to that of the river and 1500mts away from the power unit.The cost of the cable below water is Rs.15/- per meter and cost of cable on the bank is Rs.12/-per meter. Find the total of laying the cable. Ans : 1000 (24725-cost) 33. The size of a program is N. And the memory occupied by the program is given by M = square root of 100N. If the size of the program is increased by 1% then how much memory now occupied? Ans:0.5%(SQRT 101N) 34. In Madras , temperature at noon varies according to -t^2/2 + 8t + 3, where t is elapsed time. Find how much temperature more or less in 4pm to

9pm. Ans: At 9pm 7.5 more 35. The size of the bucket is N kb. The bucket fills at the rate of 0.1 kb per millisecond. A programmer sends a program to receiver. There it waits for 10 milliseconds. And response will be back to programmer in 20 milliseconds. How much time the program takes to get a response back to the programmer, after it is sent? Ans: 30 36. A man, a woman, and a child can do a piece of work in 6 days. Man only can do it in 24 days. Woman can do it in 16 days and in how many days child can do the same work? Ans: 16 37. Which of the following are orthogonal pairs? a. 3i+2j b. i+j c. 2i-3j d. -7i+j Ans: a, c 38. If VXUPLVH is written as SURMISE, what is SHDVD? Ans: PEASE 39. If A, B and C are the mechanisms used separately to reduce the wastage of fuel by 30%, 20% and 10%.What will be the fuel economy if they were used combined. Ans: 20% 40. What is the power of 2? a. 2068 b.2048 c.2668 41. Complete the series. 3, 8, --, 24, --, 48, 63. Ans: 15.35 42. Complete the series. 4, -5, 11, -14, 22, -- Ans: -27 43. A, B and C are 8 bit no's. They are as follows: A -> 1 1 0 1 1 0 1 1 B -> 0 1 1 1 1 0 1 0 C -> 0 1 1 0 1 1 0 1 Find ((A-B) u C)=? Hint: 109. A-B is {A} - {A n B} 44. A Flight takes off at 2 A.M from northeast direction and travels for 11 hours to reach the destination, which is in northwest direction. Given the latitude and longitude of source and destination. Find the local time of destination when the flight reaches there? Ans: 7 am

45. A can copy 50 papers in 10 hours while both A & B can copy 70 papers in 10 hours. Then for how many hours required for B to copy 26 papers? Ans: 13 46. A is twice efficient than B. A and B can both work together to complete a work in 7 days. Then find in how many days, A alone can complete the work? Ans: 10.5 47. A finish the work in 10 days. B is 60% efficient than A. So how many days do B takes to finish the work? Ans :100/6 48. A finishes the work in 10 days & B in 8 days individually. If A works for only 6 days then how many days should B work to complete A's work? Ans: 3.2 days 49. Given the length of the 3 sides of a triangle. Find the one that is impossible? (HINT: sum of smaller 2 sides is greater than the other one, which is larger) 50. Find the singularity matrix from a given set of matrices? (Hint det(A)==0)) 51. (Momentum*Velocity)/(Acceleration * distance). Find units. Ans: mass 52. The number 362 in decimal system is given by (1362) x in the X System of numbers find the value of X a} 5 b) 6 c) 7 d) 8 e) 9 53. Given $ means Tripling and % means change of sign then find the value of $%$6-%$%6 54. My flight takes of at 2am from a place at 18N 10E and landed 10 Hrs later at a place with coordinates 36N70W. What is the local time when my plane landed? 6:00 am b) 6:40am c) 7:40 d) 7:00 e) 8:00 (Hint: Every 1 deg longitude is equal to 4 minutes. If west to east add time else subtract time) 55. 7 8 9 11 13 ?? 19 56. there is a question like log.42=,log.43=.then log.41==?

CRITICAL REASONING:
Entirely from Barrons 12th edition 5 Model Test Papers at the end. Those who dont have barrons, you can obtain it from the Internet and get the book in a pdf format. The questions asked are from these

Model test 1: Section5 - qns 1-4 (motorist), qns 13-16 (red and brown) Section6 - qns 1-4 (conservative,democratic), qns 8-11 (latin, sanskrit), qns 12-18 (joe,larry,ned), qns 19-22 (a causes b)
Model test 2: Section1 - qns 19-22 (wallachia and rumelia) --Section6 - qns 8-12 ( ashland , liverpool), qns 13-16 (spelunker) --qns 17-22 (pesth) ---i got this one too Model test 3: Section6 - qns 1-4 (all Gs are Hs) Model test 4: Section5 - qns 8-11 (horizontal row), qns 19-22 (a,b,c cause d) Section6 - qns 8-12 (spanish, italian), qns 13-16 (all As, Bs), qns 17-22 (progressive federal party) Model test 5: Section3 - qns8-11 (museum), qns 19-22 (A is the father) Section7- qns 1-5 (prahtu, brahtu)

Here I am giving the answer of the puzzle which are from Barron 12 edition. Those people cant solve the puzzle just keep the answer in ur memory . Dont worry here all ans are correct for the puzzle. I have written the first line of the puzzle with answer. For a motorist there are three ways :ADCE Project Consolidated : ABECE Red and Brown : CBAE Letters A, B, C,D,E,F andG : BEDCCC In country X : DDBC Latin Sanskrit : DADB Joe, Larry, Ned : CCBAEBE A causes B or C but not both : DCCC Eight varsity baseball players : ADBE Delegations from Wallachia and Rumelia : AECE Byram and Adoniram : CDEA Ashalan is north of East Liverpool : BEDAC Spelunkers International : CCBB Mr. Pesth: BDEAAC On Sunday Dec 23: CDABA Airedale, Boxer, Collie: BCCADC All Gs are Hs: EECD Hotel Miranmar: DBCA Each word in a horizontal row: AEDE A B C or W may causes D: AABC Five executive of European: EADBD All As Bs Cs Ds Es Fs are Qs: EDBC Progressive federal party: CADEBB

Mrs. F official hostess: EADE Homer Meuseum: CCBB A is the father: DEEB Prahtu and Brihtu marriage: DDABE Seven varsity basket ball: DCCB
Mr.pict must: DECDB

HR: A mam took my hr. she is very cool person. i gave very cool interview to the mam she asked me the question. introduce urself?
WHY tcs??

i told tcs is the top 10 global company. tcs viges us very good tranning. & very good working environment . Previous most of the sinior recru teed by tcs ,they told me to join Tcs. hobby? i told playing cricket & searching google. she ask me have u seen yesturdays IPL match ? i told yes mam . kkr vs bangalore. dada scores 40 run .hit a great six. monoj tewary got man of the match. i asked her have u seen te match... she told no, i can't get time to see the match

Whats ur weakness? I told i can't give up my hope until i finish my job.. the she asked me so r u a hopefull person? i told yes mam. and i am hopefull thet today i will get job. after listenning she gave a little smile at me.

Exacurriculam activity?
we made a short movie. what is the massage of the movie? i told the massage of the movie is :-if u not get a job don't be frustretred , don't break ur mind one day u will be a successfull man do u know unique key? i answered. what is reffrencial integrity? i answered.

what is triggered? i told i don't know pl/sql.


Aptitude test consisted of 3 sections: three sections, Sec-1 ->English section Time-20 Mins 32 questions 10 synonyms & 10 antonyms big comprehension sentence completion with too many options in one passage. Hardly 6 synonyms n antonyms i was knowing correctly though i had learned all the previous yrs papers and barrons model test paper.It generally doesn't happen.It was my hard luck.so i preferred eliminating the options to reach 2 correct option. Sec-2->Quants section Time 40 Mins 38 questions This is the most scoring section. I got almost all questions correct in this section upto question 32.I dint see last 33-38 questions of DI before.I thought there is only 32 question but managed 2 do rest in last 2 minutes.In this section 90% questions were from previous yrs papers.So be prepared with time management.Dont do mistakes lyk me.Sec-3->Analytical Reasonong Time 30 Mins 12 questions This the most toughest section...three reasoning set each containing 4 questions(4*3=12 ques total).But to me it seemed very easy.All the sets were from GRE Barron's model test paper only with same sequence of questions.i mugged up all.So i took hardly 5 min for all 12 questions. So go with the GRE Barrons 12th edition's model test papers(all english n reasoning section). Our interview was to be held on 8th january.So i got xtra time 2 prepare for interview. I was very much nervous before going to my interview panel bt as soon as i entered room i kept myself cool.My interview started at 3.30 pm me:may i cum in sir? HR:yes me:thank u sir.shaked hands with gud after noon wish. HR:take ur seat. me:thank u sir.He then saw my tcs form which i had filled. HR:ok deepak first introduce urself. me:i told.dont go deep into ur family background or ur marks.tell abt ur gud/strong qualities, current projects etc. HR:u r having 2 yr gap.why so? me:sir,being a bright student thru out my life i was sure of cracking IIT jee after preparing for 1 yr.so dint fill other forms.i dint keep plan B that time.After that mistake i learnt one should always be ready with plan B.so i had no option other than to drop.Next yr i filled all forms n got admission in this clg thru AIEEE.With that mistake i always go with 2 options in my life now.I was trying to answer this question in company's favour only.so be prepared if u have gaps. HR:ok.....where did u do ur summer training? me:sir..in HP on Networking. HR:what things u learnt there? me:i told blah... blah... blah what came to mind that tym.. HR:can u name all the OSI layers? me:i told. HR:what is difference between hub and switch? me:i told. HR:what is ur favourite subject? me:c,c++ HR:then y u did training on networking? me:sir,i studied c++ in my +2.i dint find anything new during semester in c++.so thought of trying something new.networkin was new to me.so did training on networking. HR:ok...give me the outputs of these 2 programs? me:i gave confidently though it was wrong but i got convinced when he replied the right answer. HR:do u know any other subjects like DBMS? me:yes sir HR:what is normalization,BCNF,3NF,2NF n 1NF?which one is best? me:i told all. HR:what is multithreading n multitasking? me:i told. HR:r u gud in programming? me:yes sir HR:can u write program for string palindrome? me:yes.i wrote the whole program correctly.he saw my program's each line till 1 min n then kept aside.

HR:so u r going 2 be an engineer.must be going for GATE xam? me:no sir.i m not interested in GATE at all. HR:then must be opting for CAT ? me:no sir. HR:why?evryone is going for CAT.why not u? me:sir,i studied the BASIC language when i was in class 7 n had developed a racing game in class 9. i had thought of becoming a software engineer that time only.i took computer science as fifth paper in class 11 though evry1 was against it as it required same labour as required in PCM.but i managed all.then i took CSE branch though ECE seat was vacant in same college.And my dream of software engg is going to be fulfilled in 6 mnths,how cn i change my field all of a suuden. Engineering wud prove worthless to me in that case. HR:ok....u may go now. me:thank u sir...shaked hands saying it was nice talking to u... i was one of them when the result was declared...... Hope 2 see u in TCS..... BEST OF LUCK FRIENDS.........

1st question: So Sanjeev tell me about your school days, where you born and brought up? about your hobbies and interest. 2nd question: So you have done online shopping cart project. What was your institute name I told him IETE again he asked, "where is it?" I replied, "at salt lake sector v Kolkata". 3rd question: Tell me about your project. I explained 4th question: So you have done your project on J2EE wt is J2EE? I replied. 5th question: Wht part of project you have done? I answered. 6th question: As you have also done coding in your project cd you write java code to connect with database he gave me pen and page I wrote the appropriate code. 7th question: Tell me what are pointers? 8th question: What is const pointer? 9th question: Write the logic to swap two numbers without using third variable as i start by writing a=a+b he said, just leave it its done. 10th question: Why you want to join TCS? I said," sir I am from Bihar where no one is aware about Infosys or CTS etc about brand TATA is known to all so if I will get a job in TCS then it would be very proud moment for me and my family as well. A part from that I have heard from my senior that TCS provides world class training to freshers and I objective to job also matches with the TCS envirnoment. 11th questions: Then he again asked that you have done codin in jsp in your project so tell me what are the implicit object in JSP? I told him there are 9 named 4 object and then I said I forgot the name of rest. 12th question: Tell me what os oops? 13th question: What is data abstraction in Java? 14th question: What is polymorphism? Explained with method overloading and overriding 15th question: What is late binding? answered 16th question: Show me the logic for the traversal of the Binary tree?

Than he was watching my CV and told you are adventurous person. Then he asked you have very good marks in B. Tech but why your 10th and 12th marks are not up to the mark? I told him sir this is scenario of Bihar board. He also asked about my AIEEE rank. Then he came to DBMS 17th question: Do you know Normalization? Yes sir 18th question: What is 3rd normal form? how to break in 3nf show me example Answered with eample 19th question: What is Outer join? I said sir I have studied but I forgot. 20th question: Where we use where clause and having clause? answered with example I have tried to cover all questions asked to me. I was lucky that I was asked questions from my interested subject only. But this was not trend for all students. Some of my friends get full out of context questions. * Some were asked only HR questions * Some were asked only tech questions * Some were asked only project questions One friend was asked this is your tie (taking his tie in his hand) compare it with TCS? Abhishek (my friend) was good at all sub like C, Java DS and DBMS but he was asked question from Unix So study only basics. Hope for the best and prepare for the worst. At last lots depends upon your luck Timer will be displayed on the top right corner of the screen set 1 1) Form meaningful word from given letter: ARPAEEKT and tell meaning of that word? a) The birdTHE BIRD b) The animal Ans: The bird -parakeet 2) There is a lab asistant. Who went 1 mile to North. 1mile to East. killed a bear. Returned to lab. What is the colour of the bear? a) White b) Black c) Blue d) Multicolor Ans: White (Remember the answer of 1st two questions) 3) A pony ran away from stable she went to new york then went to US then to Mumbai ran through grass. Horse cought her after 4 hours then pony went to sky. If speed of horse is 35 kmph and he left stable 3 hours after pony left stable then what is speed of pony? Ans: Pony ran for total 7 hours

Same dist covered by horse in 4 hrs with 35 kmph Thus dist of horsr 35X4 Speed of pony-(35X4)/7 4) Mumbai building dept decide to give numbers to building. 1-30 numbers were given to industrial buildings. 1-100 were given to res.buildings. Find how many number of 2s were used while numbering res.buildings(2 number kiti vela aala?) a) 18 b) 19 c) 20 d) 21 Ans: 20 5) Find (31*31*31/44*44*44)*(87*87*87/56*56*56) Use calculator 6) A toy train can make 10 sounds sound changes aftr every 4 mins, now train is defective and can make only 2 sounds. Find probability that same sound is repeated 3 times consecutively(1 out of )? a) 16 b) 8 c) 12 d) 4 Ans: (1/2)*(1/2)*(1/2)=(1/8) Thus 1 out of 8 7) (Some nonsense stuff) Resistance is X ohm voltage Y then wat is current Ans: V=IR 8) If a person points at a photo & say d person in dis picture is son of my fathers son. What is relation? a) Son b) Father c) Grandson d) Grandfather 9) I have 3 grandsons. Age different b/w 2 of grandsons X years 1st grandson is twice elder than younger one adiition off ages of all the three is y than what is age of eldest grandson?( There is some value in X and Y) 10) Ferrari is leading car manufacturer.*Ferrari S.p.A.* is an Italian sports car. It has enjoyed great success. If Mohans Ferrari is 3 times faster than his old Mercedes wich gave him 35 kmph, if Mohan travelled 490 km in his ferrari the hw much time(hours) he took? a) 8 b) 4 c) 7 d) 7.33 (Options may be different) 11) Lion rat stayin in jungle happily. Lion lies on: Mon, Tue, Wed RAT lies on: Wed, Thurs, Sat If lion says: I didnt lie yesterday RAT says: e1 i didnt lie yesterday So what day is today? 12) The ratio of current age of X and Y is 5:7, after hw many years der age ratio will b 7:9? 13) Inspired by fibonacci series sanket decided to create is own series which is 1, 2, 3, 7, 7,

22, 15, 67, like this, then what no come immediately before 63? Ans: 202 Expalnaton; check altenate no.1, 3, 7, 15=====>n*2+1 Similarly 2, 7, 22, 67=======>n*3+1 So series is 1, 2, 3, 7, 7, 22, 15, 67, 31, 202, 63. 14) Exactly similar to above question 15) Valentine day 14 feb 2005, was celebrated by n and u on monday, he was very happy, he n she then day on 14 Feb 2010? (Similar to dis some date questions was there) 16) By using 1, 2, 3, 4, 5, how many 5 digit no. Can be formed which is divisible by 4, repetation of no. is allowed? 17) The cost 1 plum is 1 cent, 2 apples is 1 cent, 3 banana is 1 cent. If Rahul buys same amount of fruits for his 3 sons spending 7 cent den what amount of fruit each child will get? Ans: 1 plum, 2 apple, 1 banana Explanation: 7/3=2.333 cents for each child According to ans given for the sum each child will get 1 plum, 2 apple, 1 banana 18.2880 is divided by which smallest no. so we get no. which is perfect square? Ans= 5 Explanation 2880/5=576 19) There are to prime no (with some nonsence stuff) then addition of two prime no is 13, n multiplication is 21, then what are some of there squares? Explanation: XY=21 and X+Y=13. Solve using calci Ans of X & Y will be in points then x2+y2=? 20) Smita was makin 1 design (again some nonsence) size of larger cube to be made is 5*5*5 using smaller cubes of 1*1*1. She created solid larger cube then she decided to make hollow cube then how many 1*1*1 cubes required to make hollow larger cube Ans: 104 Explanation: (25+25)+(15+15)+(12+12)=104 21) 2X/5Y=5X/3Y then wat is x/y (some easy some) 22) A pizza parlor provides pizzas there were 2 toppings available initially peperoni and salami. But now they, ve introduces 8 new toppings (some names) to select from a person wishes to buy two different pizzas of new topping in how many ways he can do that? Ans: 8 X 7=56 23) Person travels to (some nonsence stuff). If he goes from A to B with speed of 4kmph and returns back to B with speed of 5 kmph. What is his avg. speed of journey? (values may be different) Ans: 4.44 (its not 4.5) Explaination : 2PQ/(P+Q)===2*4*5/(4+5)=4.44kmph 24) There is a dice having value from 1. 6 on each face and a pack of cards having face card aces (hugh chunk of nonsense). When 2 dies are thrown and their scores are added then

which sum will come max number of times? a) 8 b) 9 c) 10 d) 11 Ans: 8 Explanation : 8-2,6 3,5 4,4 9-4,5 3,6 10-5,5 4,6 11- 5,6 Thus 8s probability is more 25) Susha brought terilon cloth and rope to (some nonsence now just go to last 2 lines). If rope is 153 meter long and it is to be cut into pieces of 1 meter long then how many times will she have to cut it? Ans: 152 times 26) (don't remember the exact questions but procedure was something luck this). 8th year 1/1024,, 9th year1/512, 10th year1/256 then after how many years 1/32? Ans: 13 27) Some questions on this formula(a3-b3) & (a+b)^2=a2+b2+2ab 28) There are 2 cans A and B one of Milk and other of Water resp., both of same quality first one teaspoon of milk from A can was added to B can then one teaspoon from B can was added to A can then wich of the folloe\wing is true. 1. Can A contain more milk than water in can B 2. Can A contain less milk than water in can B 3. Both contain same qty of milk and water Ans: Option 2 29) Probability sum on gloves, jackets and something luck that (moderate sum) 30) There are some 2 wheelers and 4 wheelers parked (some nonsense). Total number of wheels present is 198(some value don't remember exact one) then how many 4 wheelers wer there (i don't remember whole sum exactly) Ans: The equation will be 2x+4y=198(the total number of wheels given) not put the values in the options to find out

Total candidates:236 (nearly) Cleared written test:140 Finally selected: 58 The paper as usual consisted of 3 sections::::
1) VERBAL (10 antonyms,10 synonyms, fill in the blanks comprehension) 2) APTITUDE (38 questions ) 3) CRITICAL REASONING ( 12 questions )

Section1: For syno.& anto. just go through their previous papers(GRE BARRONS `12th edition will help u a lot for both)...im giving some examples :

Syno : claustrophobia, baneful, precarious etc. Anto : fledgling, sumptuous, begrudge(for this part my suggestion is to go through all the anto given in BARRON exercise)

Comprehension: that was very easy, I have done 6 sure out of 12. But others faced problem in solving dat part, as papers were different.
Section2: It was real scoring ,everybody has solve at least 33 out of 38. I solved 30 correctly .Be very careful in solving dis part as u have to score ..... It was very too. Just go through all the papers of previous years....they will only change the data, thats it. We were informed 2 days ago ,so we did not have enough time .... I just solved the previous papers (from freshersworld).. and succeed. Here is the paper:(data are changed ) 1) Fill in the missing number in the sequence : 26,19,17,13,11,?,8,7 2) In the following series, how many Ws are there such that each W is followed by an C next to it if the C is not followed by a S next to it?(like this ,I cant remember mine) W C W S Q M W C S M C W C C W Q M W W C Q W C S C W A M C W C M (a)3 (b) 5(c)6 (d)4 (e)7 3) The three numbers in brackets represent the length of the sides of a triangle. Which of these does not represent a proper triangle? (a)(2m,3m,4m) (b)(1m,2m,4m) (c) (3m,4m,5m) (d) (3m,3m,3m) (e) (5m,3m,5m) 4) My flight takes of at 2am from a place at 18N 10E and landed 10 Hrs later at a place with coordinates 36N70W. What is the local time when my plane landed? 6:00 am b) 6:40am c) 7:40 d) 7:00 e) 8:00 Sol) The destination place is 80 degree west to the starting place. Hence the time difference between these two places is 5 hour 20 min. (=24hr*80/360). When the flight landed, the time at the starting place is 12 noon (2 AM + 10 hours). Hence, the time at the destination place is 12 noon - 5:20 hours = 6: 40 AM 5)A file is transferred from one location to another in 'buckets'. The size of the bucket is 10 kilobytes. Each bucket gets filled at the rate of 0.0001 kilobytes per millisecond. The transmission time from sender to receiver is 10 milliseconds per bucket. After the receipt of the bucket the receiver sends an acknowledgement that reaches sender in 100 milliseconds. Assuming no error during transmission, write a formula to calculate the time taken in seconds to successfully complete the transfer of a file of size N kilobytes. (n/1000)*(n/10)*10+(n/100)....as i hv calculated...~~!not 100% sure 6)If G(0) = -1 G(1)= 1 and G(N)=G(N-1) - G(N-2) then what is the value of G(6)? ans: -1 bcoz g(2)=g(1)-g(0)=1+1=2 g(3)=1 g(4)=-1 g(5)=-2 g(6)=-1 7)A, B and C are 8 bit no's. They are as follows: A -> 1 1 0 0 0 1 0 1 B -> 0 0 1 1 0 0 1 1 C -> 0 0 1 1 1 0 1 0 ( - =minus, u=union) Find ((A - C) u B) =? To find A-C, We will find 2's compliment of C and them add it with A, That will give us (A-C) 2's compliment of C=1's compliment of C+1 =11000101+1=11000110 A-C=11000101+11000110 =10001001 Now (A-C) U B is .OR. logic operation on (A-C) and B 10001001 .OR . 00110011 The answer is = 10111011, Whose decimal equivalent is 187 8)The size of a program is N. And the memory occupied by the program is given by M = square root of 100N. If the size of the program is increased by 1% then how much memory now occupied ? Sol) M=sqrt(100N) N is increased by 1% therefore new value of N=N + (N/100) =101N/100 M=sqrt(100 * (101N/100) ) Hence, we get M=sqrt(101 * N) 9)In two dimensional array X(7,9) each element occupies 2 bytes of memory.If the address of first element X(1,1)is 1258 then what will be the address of the element X(5,8) ?ans.1338 10)The temperature at Mumbai is given by the function: -t2/6+4t+12 where t is the elapsed time since midnight. What is the percentage rise (or fall) in temperature between 5.00PM and 8.00PM? 11)car is filled with four and half gallons of oil for full round trip. Fuel is taken 1/4 gallons more times in going than coming. What is the fuel consumed in coming up 12)One rectangular plate with length 8inches, breadth 11 inches and 2 inches thickness is there. What is the length of the circular rod with diameter 8 inches and equal to volume of rectangular plate? 13)what is the largest prime number in 8 digit number? (be careful bout dis type of ques they will surely give dis type of ques) What is d largest prime number that can be stored in a 9 bit register? Sol): 509(example) 14). a)Select the odd one out a. SMTP b. WAP c. SAP d. ARP b)Select the odd one out a. Oracle b. Linux c. Ingress d. DB2 c ) Select the odd one out a. WAP b. HTTP c. BAAN d. ARP 15)If A, B and C are the mechanisms used separately to reduce the wastage of fuel by 30%, 20% and 10%.What will be the fuel economy if they were used combined 16)Which of the following highest Standard deviation? a).5,0,5,0,5,0 b) 5,5,5,5,5,5 c) 5,5,5,5,0,5 d) -5, 5, -5, 5,-5, 5 Sol): b 17)A power unit is there by the bank of river of 750 meters width. A cable is made from power unit to power a plant opposite to that of the river & 1500 mts away 4m d power unit. The cost of cable below water is Rs 15/- per meter & cost of cable on the bank is Rs 12/-. Find d total cost of laying?(dis one is also very common ) 18)Which shape will be obtained by using these values of X & Y? XY 0 0.00001 10 1.02 100 1.72 1000 3.00 9999 4.72 Sol): Y= log10(X)

19)1. Mammal - cow ---> a. A type of 2. Snake - reptile ---> b. A part of 3. Roof - Building ---> c. Not a type of 4. Mushroom - Vegetables ---> d. A superset of Sol): 1-c, 2-d, 3-b, 4-a 20)Find d value of @@+25 - ++@16, where @ denotes square and + denotes square root? Sol): 621(like dis) 21)The size of a program is N. And the memory occupied by the program is given by M = square root of 100N. If the size of the program is increased by 1% then how much memory now occupied? Sol): 0.5% 22)Find the result of the following expression- M (363, 5) +R (5.8) +T (7. 7)-R (3.8) if, M denotes modules operation, R denotes round-off, T denotes truncations? Sol): 19 23)Find d unit of given expression - (momentum*velocity)/ (Acceleration*distance)? Sol): Mass 24)Which is not a side of triangle? a) (2,3,6) b) (3,2,4) c) (3,4,5) d) (3,3,3) Sol): b (dis type) 25)What is the power of 4? 4096 26)no.of vertices,edges, faces are given.which one among these best suits for the solid planner cube. i can't remember rest of ques. their r also bar graph,pie graph ques .... for this R>S> agarwal is enough.

Section3: this was critical reasoning section: all from BARRON"S 12th ed. model test3: 19-22(hotel miramar) model test2:17-22(mr. Pest,foreman for BUDA const.) their was another one i can't remember dat.i think it was not from BARRON... but dat one very easy too. i've done all of dem correctly...dat part was also very scoring.........

{NOTE: in TCS i don't think u have to do a lot.u can prepare within 1 days,as i got....... very easy to crack in TCS if u cleared WRITTEN. in interview they will only see your confidence,your, speaking ability,smartness,way of answring......... even if u can't ans their technical dat doesn't matter....... make sure daqt u r confident. but u need not be worry bout dat,even if u r not confident dat time they will make u to feel confident. they are very "FRIENDLY"}

TECHNICAL INTERVIEW: there were 2 interviewer. me: good afternoon sir. sir: plz have a sit. me: thank u sir. sir: sourav tell me bout yourself. me: answered sir:. (very impressed) sir: sourav u have mentioned u r the only son of ur parents, so will they permit u to go out of kolkata? me: answered. they asked several more ques on HR sir: what do u know about computer? me: i was starting to tell about language but the want the basics of computer...den i told dat one sir: (impressed) there were several more ques like:what is synchronigation?,dia of J-K flip flop,digital filter design,linked list etc {NOTE: for ece plz study the basics only. c,data structure are enough for us... plz make sure u r confidently ans every ques even if ur wrong just say with confident dat u r sure. they will prefer ur confidence rather than ur correct ans} The result was declared 2 days latter....... 58 students were selected.dat was the the best moment for me,as i was one of them...... from ECE 17 selected. Thank you.see you in TCS............................ There will be three section 1.English 32 ques 20min 2.Aptitude section 38ques 40 min. 3.critical reasoning 12ques 30min. 1.ENGLISH SECTION: PLZ HAVE A LOOK ON THESE:(I was not prepared for this section,I was very much prepared for the other two sections.I have given all ans as b or c in this section)

We had 10 synonyms and 10 antonyms For each of the words in Capital letters, choose from among the answers, the word that is closest in meaning:

Q No. 1 GARRULOUS (a) Arid (b) Hasty Q No. 2 INTER (a) Bury (b) Amuse

(c) Sociable (c) Relate (c) Distant (C)dishonorable

(d) Quaint (d) Frequent (d) Economical (D)adventurous

(e) Talkative (e) Abandon (e) Fantastic

Q No. 3 CHIMERICAL (a) Developing (b) Brief Q No. 4 COPIOUS (A) plentiful (B)cheating

(E)inspired

Q No. 5 EQUANIMITY (a) Clamour (b) Disparity Q No. 6 GALLEON (a) Liquid measure (b) Ship Q No. 7 REGAL (a) Oppressive Q No. 8 MUSTY (a) Stale (b) Royal (b) Necessary

(c) Composure (c) Armada (c) Major (c) Indifferent (c) Obeisance (c) Experience

(d) Propensity (d) Company (d) Basic (d) Nonchalant (d) Insult (d) Anxious

(e) Indivisibility (e) Printers proof (e) Entertain (e) Vivid (e) Message (e) Sallow

Q No. 9 INDIGNITY (a) Pomposity (b) Bombast Q No. 10 CANDID (a) Vague (b) Outspoken

For each of the words in capital letters, choose from among the answers, the closest word that has the opposite meaning: Q No. 11 REPUDIATE (a) Hesitant (b) Neutrality Q No. 12 CREDIT (a) Believe false (b) Treat as equal Q No. 13 TENACIOUS (a) Fast-running (b) Intention Q No. 14 PRECIPITATE (a) Intricate (b) Devious Q No. 15 QUIXOTIC (a) Slow (b) Abstemious (c) Admit (d) Polite (e) Incongruity

(c) Make more difficult (d) Underemphasize (e) Irresolute (c) Obnoxious (c) Posthumous (c) Pragmatic (d) Holding fast (d) Dilatory (d) Benevolent (e) Irresolute (e) Contradictory (e) Grave

Q No. 16 ADMONITION (a) Premonition (b) Hallucination (c) Escape Q No. 17 PROCLIVITY (a) Prodigality (b) Avoidance (c) Credence

(d) Commendation (e) Trepidation (d) Calumny (e) Inception

Q No. 18 OMNISCIENCE (a) Power (b) Extravagance (c) Magnanimity Q No. 19 PRECIPITIOUS (a) Cooperative (b) Cautious Q No. 20 PROTRACT (a) Defy (b) Supplement (c) Inaccurate (c) Postpone

(d) Conscience (e) Ignorance (d) Formal (d) Shorten (e) Simplistic (e) Design

There was also a passage with 6 questions and fill in the blanks with 6 questions in this section.

Quantitative Aptitude Section : Q1. Fill in the missing number in the sequence 7 8 9 11 13 ?? 19 26 (c) 17 (d) 15 (e) 18

(a) 24 (b) 21

Q2. In the following series, how many Us are there such that each U is followed by an G next to it if the G is not followed by a S next to it. U G U S Q M U G S M G U G G U Q M U

(a)2 (b)3 (c)5 (d)4 (e)6 Q3. If FLPMXQHO is coded as COMPUTER how will GFVMOXB be coded? (a) PROCESS (b) CENTRAL (c) PLEASE (d) SURMISE (e) DISPLAY Q4. Suppose the first and second letters in the word CONSEQUENCES were interchanged, also the third and fourth letters, the fifth and sixth etc. Print the letter that would then be the eighth letter counting from the right. (a) C (b) E (c) N (d) Q (e) S Q5. How would the decimal number 362 be represented in a base -6 number System? (a) 1234 (b) 1342 (c) 1362 (d) 1544 (e) 1645 Q6. What is the largest prime number that can be stored in a 7-bit word computer? (a) 123 (b) 129 (c) 121 (d) 127 (e) 119 Q7. If n = 15 x 28 x 26, which of the following is NOT an integer? a. n / 15 b. n / 21 c. n / 64 d. n / 35 e. n / 78 Q8. Which of the following is a power of 2? a. 2148 b. 2096 c. 2048 d. 2444 e. 2198 Q9. Pick the odd one out:

(a) ORACLE (b) SYBASE (c) SMTP (d) DB2 (e) INGRESS

Q10.

Pick the odd one out:

(a) UNIX (b) WINDOWS NT (c) LINUX (d) MVS (e) C++ Q11. The three numbers in brackets represent the length of the sides of a triangle. Which of these does not represent a proper triangle? (a) (5m,3m,4m) (b) (4m,5m,4m) (c) (4m,6m,8m) (d) (6m,2m,2m) (e) (12m,6m,8m) Q12. The three numbers in brackets in each of the following options represents the number of edges ,the number of vertices and the number of faces respectively. Find out which of these represents a solid planar cube? (a) (4,8,12) (b) (12,8,6) (c) (8,6,12) (d) (8,8,8) (e) (12,6,8) Q13. Which set of data exhibits a higher Standard Deviation? (a) 3, -3, 3, -3, 3, -3 (b) 3, 0, -3, 3, 0, -3 (c) 3, 3, 3, 0, 3, 3 (d) 3, 3, 3, 3, 3, 3 (e) -3, -3, -3, -3, -3, -3 The three circles below represent the number of people speaking French, German and English. Answer the next three questions based on the diagram(There was a ven diagram. This is very easy section. I have tried a lot to give u the diagram below)

---> German

* * * * * * * * * 45 * 14 * 49 * * * * * * * 12 * * * * * * * * * * * 13 * 16 * * * * * * * * * * 51 * * * * * * *

<---English * * * * *

<----French

Q14. How many more (or less) people speak English than people who speak German? (a) 2 (b) 3 (c) 1 (d) 5 (e) 6 Q15. What percentage of people speaking French can also speak German but not English? (a) 16 (b) 14 (c) 17.4 (d) 15 (e)13 Q16. What percentage of people can totally speak all three languages? (a) 4 (b) 5 (c) 6 (d) 7 (e) 8

The figure on the left represents number of members in a Club and the figure on the right depicts profession wise distribution for 1998

Members (in "00s)Here was a bar chart .This is also very easy section.

Q17. Which year has registered the maximum percentage growth in members? (a) 1996 (b) 1998 (c) 1994 (d) 1686 (e) 1888 Q18. What are the average members for 1995-1999? (a) 7700 (b) 8800 (c)6800 (d)4800 (e) 7800 Q19. If 10% of members in 1998 left the Club, how many fresh members were made in 1999? (a)4500 (b) 4900 (c)4600 (d) 4400 (e) 5400 Q20. A hypothetical physical quantity is defined as (Energy X Time X Time) (Mass X Distance) In what fundamental units would this quantity be expressed? (a)Time (b) Distance (c) Velocity (d) Mass (e) None Q21. A, B and C together can finish a piece of work in 4 days; A alone can do it in 12 days and B alone can do it in 18 days. How many days will be taken by C to do it alone? (a)21 (b)27 (c)9 (d)6 (e)8 Q22. Which of these matrices is singular A=|8 6| B=|-1 7| C=|3 2| D=|0 3| |4 3| |0 6| |5 9| |3 0| (a)A (b)B (c)C (d)D (e) None Q23. Match the following relationships: (i) Animal Lion (1) Not a type of (ii) Television Conveyance (2) Part of (iii) Square Polygon (3) A type of (iv) Piston Engine (4) Superset of (a) i-3,ii-1,iii-4,iv-2 (b) i-4,ii-1,iii-3,iv-2 (c)i-4,ii-1,iii-2,iv-3 (d) i-3,ii-2,iii-1,iv-4 (e) i-1,ii-4,iii-3,iv-2 Q24. If @ stands for squaring and + stands for square root what is the value of @ @ + (25) - + + @ (36)? (a) 625 (b) 631 (c) 619 (d) -619 (e) 627 Q25. A sequence is defined recursively as g(0) = -1; g(1) = 1 g(n) = g(n-1) + g(n-2) What will be the value of g(5)? (a)1 (b)-2 (c)3 (d)2 (e)5

Q26. What curve best suits the following data: XY 0.99 0.00001 10.04 1.02 99.98 1.997 1000 3.0 9990 4.004 (a) y = logn x (b) y = log10 x (c) y = ex (d) y = -log10 x (e) y = - ex

Q27. A Two-dimensional array X (9,7) is stored linearly column-wise in a computer's memory. Each element requires 8 bytes for storage of the value. If the first byte address of X (1,1) is 3000, what would be the last byte address of X (8,5)? (a) 3241 (b)3541 (c)3431 (d)3556 (e)3224 Q28. Of the four straight lines A,B,C,D find out which pair forms an orthogonal set A: 3x+4y-7 = 0, B: y = -x , C: y = 7x+3, D: 4x = 3y+5 (a)AD (b)AC (c)BC (d)BD (e)AB Q29. Evaluate the expression M(843,5) + R(5.8) + T(7.7) - R(3.4) where M stands for Modular arithmetic, R stands for Round-off operation and T stands for Truncation Operation (a)12 (b)11 (c) 13 (d)14 (e)15 Q30. Three independent strategies A, B and C have been initiated for cost cutting in a company producing respectively 20%, 30% and 10% savings. Assuming that they operate independently, what is the net saving achieved? (a) 45% (b) 49.6% (c) 54% (d) 56% (e) 48% Q31. The log values of two numbers to the base 10 are given as below: X log10X 3.142 0.4972061807 3.143 0.4973443810 Find log103.141. (a) 0.49706 (b) 0.59720 (c) 0.49110 (d) 0.49420 (e) 0.49440 Q32. What equation best describes the curve shown below:(there was a curve) (a)y = tan x (b) y + 3 = x (c)y + x = 0 (d) y = ex (e) y = Cos x

Q33. The temperature at Kochi is given by the function: -t2/6+4t+12 where t is the elapsed time since noon. What is the percentage rise (or fall) in temperature between 6.00 PM and 9.00 PM? (a) 15% (b) 20% (c) 25% (d) 10% (e) 10.5% Q34. An aircraft takes off from A (72o N Lat, 40o E Long) at 2.00 AM local time to B (32o N Lat, 50o W Long). If the flying time i s 8 hours what is the local time of landing at B? (a)5.40 AM (b) 3.00 AM (c) 4.00 AM (d) 5.00AM (e) 8.00AM Q35. Fill in the last row of the following Truth Table: A00001111 B00110011 C01010101 (A B ) C Interpret the resulting bit pattern as an integer in an 8-bit computer and write the decimal value. (a) 127 (b)129 (c)128 (d) 131 (e)122 Q36. A file is transferred from one location to another in 'buckets'. The size of the bucket is 10 kilobytes. The bucket gets filled at the rate of 0.0001 kilobytes per millisecond. The transmission time from sender to receiver is 10 milliseconds per bucket. After the receipt of the bucket the receiver sends an acknowledgement that reaches sender in 100 milliseconds. Assuming no error during transmission, write a formula to calculate the time taken in seconds to successfully complete the transfer of a file of size N kilobytes. (a) 10.11 N (b) 10.011 N (c) 11.01 N (d)11.011 N (e)10.101 N

Q37. The productivity of a group of workers is given by the function P(N) = 4000 N, where N is the total strength. Find the percentage change in productivity if the strength of the group is increased by 1%. (a) 0.75% (b) 0.5% (c) 0.25% (d) 1% (e) 2% Q38. A power cable is to be run from a power plant on the bank of a river 900 meters wide to a factory that is located 3000 meters downstream on the opposite bank. If the cost of laying cable under water is Rs. 5 per meter and that of laying overhead on land is Rs. 4 per meter, find the point downstream where the cable is to cut across the river. (a) 450 (b) 3900 (c) 1800 (d) 2100 (e) 2700

Please try the above questions own. I had correctly done all these questions. CRITICAL REASONING: Entirely from Barrons 12th edition 5 Model Test Papers at the end. Those who dont have barrons, you can obtain it from the Internet and get the book in a pdf format. The questions asked are from these Model test 1: Section5 - qns 1-4 (motorist), qns 13-16 (red and brown) Section6 - qns 1-4 (conservative,democratic), qns 8-11 (latin, sanskrit), qns 12-18 (joe,larry,ned), qns 19-22 (a causes b) Model test 2: Section1 - qns 19-22 (wallachia and rumelia) --Section6 - qns 8-12 ( ashland , liverpool), qns 13-16 (spelunker) --qns 17-22 (pesth) ---i got this one too Model test 3: Section6 - qns 1-4 (all Gs are Hs) Model test 4: Section5 - qns 8-11 (horizontal row), qns 19-22 (a,b,c cause d) Section6 - qns 8-12 (spanish, italian), qns 13-16 (all As, Bs), qns 17-22 (progressive federal party) Model test 5: Section3 - qns8-11 (museum), qns 19-22 (A is the father) Section7- qns 1-5 (prahtu, brahtu), qns 21-25 (scurvy)

I got the "Latin,sanskrit","Some series" and "progressive federal party" . If you practice well the Barron paper then it will take 2-3 minutes to complete this section.I had done it within 5 minutes. Aptitude was very well for me and was confident that i'll clear the apti. When the apti result decleared i was the second one in our 60 students batch.(60 students per batch .4 batch was there in our college.)

I was waiting for interview .It was 5.00 PM when i went for interview. HR+TECH INTERVIEW: For technical round, get well dressed in formal outfits . Bring the file with ur certificates and Resumes. Dont bluff in the resumes, as they check your resume and ask questions related to it. INT: Get in ME : Good afternoon,Sir.Shuted down the door. INT: So u r Soumyadeb Halder,sit down . ME : Thank you sir INT: (watched very carefully the TCS-form which i filled up.) Tell me about your final year project. ME : i told and explained our project. INT: what r u going to implement in your project. ME : answered. INT: (watched very carefully my c.v.)asked what is DBMS? ME : Explained. INT: what is the definatin of Database management System? ME : answered. INT: How to add a column in a table? ME:I can not remember the query but i can explain it. INT:Explain. ME: Explained. INT: What is DML? ME: Aswered. INT:How to retrieve a value from a table? ME:Answered. INT: What is call by value and call by reference in c? ME:Answered. INT:I am going to update a value then what is neede call by value or call by reference? ME : call by reference sir. INT Why? ME: Explained. INT:(wrongly writen a c program and gave that to me)What will be the output? ME:First caught the wrong portion and then answered.(he was very much impressed) INT:Do you have any question? ME:Asked a question with a smile. INT:Answered. ME: Thank you sir. Thats all in my interview(there is no HR question in my interview.some of my friends faced a lots of HR questions).

At the last result was declared at 9.00 PM.I was very very happy that i was one of the 115 selected students and very much thankful to freshersworld for giving me the right approach to crack TCS.

Wish You "ALL The Best".See you in TCS. Soumyadeb Halder.

The paper was divided into 3 sections 1) ENGLISH, 2) QUANTITUDE, 3) LOGICAL REASONING -Puzzle part.. It was an online test so u didnt have the choice to choose a part in which u r comfortable in,u have to start with English, then Quant and then Puzzle. For ENGLISH see the ANTONYM given in (Barron's Gre (12th edition) Chapter-6 Antonym Exercise) some of the words I can remember are 1) GAUCHE- lacking social polish, 2)DISCREETNESS- prudence, 3)CHURLISH- rude There were also couple of passages(where u have to fill the missing sentences) which were a bit tricky so I will advice u guys to solve that first and den come to Synonyms and Antonyms (which are 10 each in total) For QUANT some of them were quite easy such as 1) Interchange MEASUREMENTS the first with 2nd letter, 3rd with 4th and so on., and den get the 12th letter from right Ans :) E 2) There was a table where for certain values of X., there were some values of Y Ans :) y=log(x) 3) How many zeros are there in 100! Ans ;) 24(get LCM of 100 and add the no.s) 4) the sides are given which triangle is not possible.. (A) (2,3,6) (B) (3,2,4) (C) (3,4,5) (D) (3,3,3) (E) (5,3,5) Some other Qs were: 5) Select the odd one out a. Oracle b. Linux c. Ingress d. DB2 <!--[if !supportLineBreakNewLine]--> <!--[endif]--> 6) Select the odd one out a. SMTP b. WAP c. SAP d. ARP 7) Select the odd man out. a. Java b. Lisp c. Smalltalk d. Eiffel

Some of the other questions were exactly of the same pattern like previous years papers (Datas are changed of course)

Say it finding the no.s from one base to another as in NUMBER SYSTEM, STANDARD DEVIATION, PROFIT AND LOSS, PERCENTAGE, PROBABLITY, TIME AND WORK, NUMBERS, AVERAGE.AGES, RATIO AND PROPORTION, ALLIGATION OR MIXTURE, SIMPLE INTEREST, COMPOUND INTERSET PERMUTATION & COMBINATION etc For PUZZLE part it was exactly from Barron's Model Test papers (Datas not changed)..it was basically from model test papers. Model test 1- Section 5: 1-4, 5-6, 8-12, 13-16, Section 6: 8-11, 12-18, 19-22 Model test 2-Section 1: 1-4, 12-18, 19-22, Section 6: 8-12, 13-16, 24-25 Model test 4- Section 5: 8-11, Section 6: 8-12 Model test 5- Section 3: 1-4, 8-11, 23-25, Section 7: 1-5, 6-9, 16-20, 21-25 When I cleared my aptitude I was quite relieved Then I went for my interview at around 8.00pm HR and TECHNICAL was taken togetherfirst of all I was made to wait for around 7 min outside the room which was quite nerve wrecking and then it started Me:) Good Evening Sir Interviewer) Nodded and asked me to sit Interviewer) Tell me about urself Me) Told with poised Interviewer) asked me about my favourite subject Me) Digital Electronics Interviewer) OK tell me the diff between a digital and analog signal Me) explained with diagrams Interviewer) What is the discrete value of a digital system Me) 0 and 1 Sir Interviewer) OK tell me where u do u see yourself in next five years Me) Told Interviewer) what kinds of transistors are there Me) about to say but another sir came in and interrupted me and den he asked Interviewer) What do you mean by AM and FM Me) Explained Interviewer) have u seen a radio and how we have to tune itwhat exactly happens: Me) explained with lan Interviewer) Do you know about Mobile Communication Me) A bit sir but not in details as I will study this in the next semester Interviewer) Do you know HLR or something Me) I didnt understand as it was from Mobile Communication but I told him about CDMA technique coding, PCM etc So he got impressed and asked me Interviewer) OK u know ur subject lets go into programming language Which language u know ? Me) C

Interviewer) OK what is meant by local, global, static variable, what is static global variable (everything abt storage class) Me) explained thoroughly Interviewer) Do u know pointers Me) Yes sir Interviewer) Take two pointers string and concatenate them Me) I did it with strcat() Interviewer) very smart Interviewer) Do it without using functions Me) I tried.. but couldnt get it Interviewer) then asked me to declare integer and assign an integer pointer and then print the post increment of it and get the output Me) done Interviewer) do you why have we asked u to fill the languages column Me) said because of some preferences and topographical restrains Interviewer) was satisfied and .shook hands When the result was declared I felt like floating in the sky it was an out of this world kind of feeling My advice will be to all aspirants is that back ur knowledgeur ability and belief in urselftry to get the first few lines in the interview to be perfect and rest u will sail through. NOTE: For Non CSE students my advice is to know ur core subjects properly and hav only sound knowledge of C, data structures etc. Hope to meet in TCS.Have a nice day Regards Section 1: VERBAL ( 32 Questions ,20 minutes ) Directions for questions 1-10:Find the synonyms of the following words 1. ADAGE A. advice Ans: B 2. TO DISPEL A. to dissipate Ans: A 3. ERRATIC A. unromantic

B. proverb

C. enlargement

D. advantage

B. to dissent

C. to distort

D. to disfigure

B. unknown

C. popular

D. steady

Ans: B 4. TO MERIT A. to embrace Ans: C 5. RAPT A. lively Ans: B 6. TO HEAP A. to pile Ans: A B. to forbid C. to proceed D. to share

B. to devote

C. to deserve

D. to combine

B. concealed

C. engrossed

D. prototype

7. OVULATE A. penury Ans: C 8. Motley A. Uniform Ans: D 9. Mitigate A. palliate Ans: A 10. Compunction A. Regret Ans: D

B. immunize

C. fertilize

D. reproduce

B. homogenous

C. monochrome

D. Assorted

B. Aggravate

C. exacerbate

D. None of these

B. scruple

C. qualm

D.

None of these

Directions for questions 11-20:Find the

Antonyms of the following words

11. Moribund

A. declining Ans: D 12. Repudiate A. reject Ans: C 13. Inundate A. starve Ans: A 14. Tenacity A. obstinacy Ans: B 15. Sobriety
A. flippancy

B. dilapidated

C. waning

D. thriving

B. disclaim

C. acknowledge

D. renounce,

B. swamp

C. deluge

D. drown

B. irresolution

C. firmness

D. resolve

B. temperance

C. moderation

D.

soberness

Ans: A 16. Hidebound A. narrow-minded Ans: D 17. Nebulous A. vague Ans: C 18. Debacle A. catastrophe Ans: C 19. Candid B. fiasco C. success D. shambles, B. imprecise C. precise D. hazy B. conservative C. prejudiced D. broad-minded

A. frank Ans: B 20. Dangle A. stick up Ans: A

B. guarded

C. open

D. truthful

B. sway

C. suspend

D. droop

Directions for Questions 21-26: Read the passage and answer the questions that follow on the basis of the information provided in the passage. Educational planning should aim at meeting the educational needs of the entire population of all age groups. While the traditional structure of education as a three layer hierarchy from the primary stage to the university represents the core, we should not overlook the periphery which is equally important. Under modern conditions, workers need to rewind, or renew their enthusiasm, or strike out in a new direction, or improve their skills as much as any university professor. The retired and the aged have their needs as well. Educational planning, in their words, should take care of the needs of everyone. Our structures of education have been built up on the assumption that there is a terminal point to education. This basic defect has become all the more harmful today, A UNESCO report entitled 'Learning to Be' prepared by Edger Faure and others in 1973 asserts that the education of children must prepare the future adult for various forms of self-learning. A viable education system of the future should consist of modules with different kinds of functions serving a diversity of constituents. And performance, not the period of study, should be the basis for credentials. The writing is already on the wall. In view of the fact that the significance of a commitment of lifelong learning and lifetime education is being discussed only in recent years even in educationally advanced countries, The possibility of the idea becoming an integral part of educational thinking seems to be a far cry. For, to move in that direction means such more than some simple rearrangement of the present organisation of education. But a good begining can be made by developing open university programmes for older learners of different categories and introducing extension services in the conventional colleges and schools. Also these institutions should learn to cooperate with the numerous community organizations such as libraries, museums, municipal recreational programmes, health services etc. 21. What is the main thrust of the author ? A) Traditional systems should be strengthened B) Formal education is more important than non-formal C) One should never cease to learn

D) It is impossible to meet the needs of everyone E) There is no substitute for the extent system of education Ans: C 22. Which of the following best describes the purpose of the author ? A) To criticise the present educational system. B) To strengthen the present educational practices C) To support non-conventional educational organisations. D) To present a pragmatic point of view E) None of these Ans : D 23.Which of the following is most nearly the same in meaning as the word 'meeting' as used in the passage. A) Approaching B) Contacting C) introducing D) representing E) satisfying Ans: E 24. Which of the following is most opposite in meaning to the word 'integral' as used in the passage. A) essential Ans: B 25. According to the author, what should be the basis for awarding credentials? A) Duration of the course B) Competence of the course teachers C) Diversity of the topics covered D) Real grasp of matter or skill E) Participation in community activities B) independent C) major D) minor E) unwilling

Ans: D 26. According to the author, educational plan should attempt to A) train the people at the core B) encourage conventional schools and colleges C) decide a terminal point to education D) overlook the people on the periphery E) fulfil the educational needs of everyone. Ans : A
Directions 27-32 : Pick out the most effective word from the given words to fill in the blank to make the sentence meaningfully complete.

27. .............you meet my son in the market, ask him to come home at once A) Will Ans : C 28. Sanjay was sure..............getting a first class in the examination A) at B) of C) on D) about B) While C) Should D) Would

Ans : B 29. She seems offended .................my remarks A) with Ans : D 30. The mounting pressure was so overwhelming that he ultimately.................to her wish A) agreed in Ans : C 31. He has no objection.................my proposal A) to Ans: A B) for C) in D) towards B) cowed in C) gave in D) yielded in B) for C) upon D) at

32. Wash your hands.............water A) from Ans: B Section 2: QUANTITATIVE/LOGICAL REASONING ( 38 questions , 40 minutes ) 1. Two Pencils 8 cents 5 Pencils cost Ans: 20 cents 2. If 2x-y=4 then 6x-3y=? (a)15 (b)12 (c)18 Ans. (b) 3. Pipe A can fill in 20 minutes and Pipe B in 30 mins and Pipe C can empty the same in 40 mins. If all of them work together, find the time taken to fill the tank (a) 17 1/7 mins (b) 20 mins (c) 8 mins (d) none of these Ans. (a) 4. Thirty men take 20 days to complete a job working 9 hours a day. How many hour a day should 40 men work to complete the job? (a) 8 hrs (b) 7 1/2 hrs (c) 7 hrs (d) 9 hrs Ans. (b) 5. A goat is tied to one corner of a square plot of side 12m by a rope 7m long. Find the area it can graze? (a) 38.5 sq.m (b) 155 sq.m (c) 144 sq.m (d) 19.25 sq.m Ans. (a) 6. Mr. Shah decided to walk down the escalator of a tube station. He found that if he walks down 26 steps, he requires 30 seconds to reach the bottom. However, if he steps down 34 stairs he would only require 18 seconds to get to the bottom. If the time is measured from the moment the top step begins to descend to the time he steps off the last step at the bottom, find out the height of the stair way in steps? Ans.46 steps. 7. The average age of 10 members of a committee is the same as it was 4 years ago, because an old member has B) with C) by D) in

(d)10

been replaced by a young member. Find how much younger is the new member? Ans.40 years. 8. ABCE is an isosceles trapezoid and ACDE is a rectangle. AB = 10 and EC = 20. What is the length of AE? Ans. AE = 10. 9.Three cards are drawn at random from an ordinary pack of cards. Find the probability that they will consist of a king, a queen and an ace. Ans. 64/2210. 10. A number of cats got together and decided to kill between them 999919 mice. Every cat killed an equal number of mice. Each cat killed more mice than there were cats. How any cats do you think there were ? Ans. 991 11. If Log2 x - 5 Log x + 6 = 0, then what would the value / values of x be? Ans. x = e2 or e3. 12. The square of a two digit number is divided by half the number. After 36 is added to the quotient, this sum is then divided by 2. The digits of the resulting number are the same as those in the original number, but they are in reverse order. The ten's place of the original number is equal to twice the difference between its digits. What is the number? Ans. 46 13. A monkey starts climbing up a tree 20ft. tall. Each hour, it hops 3ft. and slips back 2ft. How much time would it take the monkey to reach the top? Ans.18 hours 14. What is the missing number in this series? 8 2 Ans. 9 14 6 11 ? 14 6 18 12

15. A certain type of mixture is prepared by mixing brand A at Rs.9 a kg. with brand B at Rs.4 a kg. If the mixture is worth Rs.7 a kg , how many kgs. of brand A are needed to make 40kgs of the mixture? Ans. Brand A needed is 24kgs.

16. A wizard named Nepo says "I am only three times my son's age. My father is 40 years more than twice my age. Together the three of usare a mere 1240 years old." How old is Nepo? Ans. 360 years old. 17. One dog tells the other that there are two dogs in front of me. The other one also shouts that he too had two behind him. How many are they? Ans. Three. 18. A man ate 100 bananas in five days, each day eating 6 more than the previous day. How many bananas did he eat on the first day? Ans. Eight. 19. If it takes five minutes to boil one egg, how long will it take to boil four eggs? Ans. Five minutes. 20.Daal is now being sold at Rs. 20 a kg. During last month its rate was Rs. 16 per kg. By how much percent should a family reduce its consumption so as to keep the expenditure fixed? Ans. 20 % 21. Can you find out what day of the week was January 12, 1979? Ans. Friday 22. From 5 different green balls, four different blue balls and three different red balls, how many combinations of balls can be chosen taking at least one green and one blue ball? Ans. 3720. 23. Three pipes, A, B, & C are attached to a tank. A & B can fill it in 20 & 30 minutes respectively while C can empty it in 15 minutes. If A, B & C are kept open successively for 1 minute each, how soon will the tank be filled? Ans. 167 minutes. 24. A person walking 5/6 of his usual rate is 40 minutes late. What is his usual time? Ans. 3 hours 20 minutes. 25. A garrison of 3300 men has provisions for 32 days, when given at a rate of 850 grams per head. At the end of 7 days reinforcement arrives and it was found that now the provisions will last 8 days less, when given at the rate of 825 grams per head.How, many more men can it feed? Ans. 1700 men

26. In the given figure, PA and PB are tangents to the circle at A and B respectively and the chord BC is parallel to tangent PA. If AC = 6 cm, and length of the tangent AP is 9 cm, then what is the length of the chord BC? Ans. BC = 4 cm 27. A boat travels 20 kms upstream in 6 hrs and 18 kms downstream in 4 hrs. Find the speed of the boat in still water and the speed of thewater current? (a) 1/2 kmph (b) 7/12 kmph (c) 5 kmph (d) none of these Ans. (b) 28. Find the smallest number in a GP whose sum is 38 and product 1728 (a) 12 (b) 20 (c) 8 (d) none of these Ans. (c) 29. If x=y=2z and xyz=256 then what is the value of x? (a)12 Ans. (b) 30. (1/10)18 - (1/10)20 = ? (a) 99/1020 (b) 99/10 (c) 0.9 (d) none of these Ans. (a) 31.A train running at 72 kms per hour crosses a coconut tree standing by the side of the track in 7 seconds. The length of the train is: (A) 104 metres Ans: B 32. If 12 men and 16 boys can do a piece of work in 5 days and 13 men and 24 boys can do it in 4 days, the ratio of daily work done by a man to that done by a boy is: (A) 1 : 3 Ans: C 33. A cistern is filled by a tap in 3 1/2hours. Due to a leak in its bottom, it takes 12 hour longer to fill. If cistern is full, how long will it take to leak to empty it? (A) 7 hours (B) 8 hours (C) 14 hours (D) 28 hours (B) 1 : 2 (C) 2 : 1 (D) 3 : 1 (B) 140 metres (C) 504 metres (D) 540 metres (b)8 (c)16 (d)6

Ans: D 34. A can do a piece of work in 7 days of 9 hours each whereas B can do the same work in 6 days of 7 hours each How long will it take to complete the work together working 8 2/5 hours a day? (A) 2 days Ans: B 35. A bus left Delhi for Ambala at 50 km/hr and turned over the same route at 40 km/hr. Thus it took 1 hour more on the return trip. The distance between Delhi and Ambala is: (A) 200 kms Ans: A 36. A man purchased a bag of rice containing 70 kgs for Rs 175. He sold it at the rate of Rs 2.75 per kg. Find the profit and loss per cent. (A) 10% profit Ans: A 37. A shopkeeper gives 3 consecutive discounts of 10%, 15% and 15% after which he sells his goods at a percentage profit of 30.05% on the cost price. Find the value of the percentage profit that the shopkeeper would have earned if he had given discounts of 10% and 15% only: (A) 53% Ans: A 38. A journey of 192 kms takes 2 hours less by a fast train than by a slow train. If the average speed of the slow train is 16 kmph less than that of a fast train, what is the average speed of the fast train? (A) 30 kmph Ans: B Section 3: CRITICAL REASONING ( 12 questions , 30 minutes)
Directions 1-12: Answer the questions given below the passage or statement as true, false or can't say.

(B) 3 days

(C) 3 1/7 days

(D) 4 2/5 days

(B) 180 kms

(C) 400 kms

(D) None of these

(B) 10% loss

(C) 12.5% profit

(D) 12.5% loss

(B) 62.5%

(C) 72.5%

(D) 68.6%

(B) 48 kmph

(C) 20 kmph

(D) 25 kmph

PASSAGE : In the past helicopters were forced to ground or crash because of the formation of the ice on the rotors and engines. A new electronic device has been developed which can detect the water content in the atmosphere and warns the pilot if the temperature is below freezing temperature about the formation of the ice on the rotors and wings. Answer questions 17-20 based on passage 1.The electronic device can avoid formation of the ice on the wings Ans.False 2. There will be the malfunction of rotor & engine because of formation of ice Ans.True 3. The helicopters were to be crashed or grounded Ans.True 4. There is only one device that warn about the formation of ice Ans.True PASSAGE : Human existence is suspicious of arbitrary divide between concise and unconcise. The concise world invades shape activity of the unconcise, while many of great activity of humanity waking as whole or partially improved by dreams. Even it could be ignored that dreams precede exceptional such a dichotomy could not be drawn as the influence of dream on waking state would remain unclear. But as yet no company rebuilt exists to record the substitute of prenatal dreaming. Answer questions 5- 8 based on passage 5. Sleepy can be creative state Ans. True 6. It is difficult to tell whether a sleeper is dream or not Ans. True 7. If we know what babies would dream about before they are born we could show that the concise and unconcise mind influence on one another . Ans. Can't say 8. It is untrue claim that concise and unconcise world never impinge one another Ans.True

PASSAGE : Although invaders represent a threat to the conservation of flora and fauna, there are two special cases in which invasion have been deliberately brought about. One is the desire to control presents by natural predators, which may have to be brought in from other countries. The second is releasing organisms into the wild (or on to farms, from which they might escape) that are completely novel, because they have been genetically engineered. There is nothing intrinsically sinister about engineered organisms, but any novelty must be regarded as a potential invader. Answer questions 9-12 based on passage 9.Pest control does not threat the conservation of flora and fauna. Ans.True 10.Genetically engineered organisms must always be regarded as potentially dangerous. Ans. False 11.Natural predators are work harmful than pests. Ans. True 12.Genetically engineered organisms escaped from the farm, they will be pose a threat to wildlife. Ans. True 1)APTITUDE TEST: Questions = 82 ; time limit = 90 minutes. no negative marking. Offline (paper & pen) test and a PSYCHOMETRY TEST also. Section 1: VERBAL ( 32 Questions ,20 minutes )
Directions for questions 1-10:Find the synonyms of the following words

1.

Depreciation A. appreciation
Ans: B

B. Deflation

C. rise

D. None of these

2.

Circumspect A Condition
Ans: C

B Inspect

C. Cautious

D Reckless

3.

Abysmal A. Slight
Ans: D

B. Deep

C. Illustrious

D. Terrible

4.

Diligent
A. hardworking Ans: A B. delinquent C. neglectful D. remiss

5.

Vehement A. Passionate B. Confess


Ans: A

C. Noisy D Moqulis

6.

Impetus A. Connect
Ans: C

B. Crucial

C. Stimulus

D Immediate

7.

Acronym A. Abbreviation
Ans: A

B. Similar

C. analogous

D. correspondent

8.

Disseminate A. Forecast B. Spread C Barns


Ans: B

D. unextended

9. Harbinger A. Naval
Ans: C

B. Uncommon C. Fore Runner

D. Glory

10. Ponderous A. light Ans: C Directions for questions 11-20:Find the Antonyms of the following words B. cumbersome C. interesting D. None of these

11) Tractable A. Objectionable Ans: A 12) Covert A. Manifest

B. Enjoyable

C. Adaptable

D. Obstinate

B. Invisible

C. Scared

D. Alter

Ans: A 13) Pensive A. Repentant

B. Sad

C. Thoughtless

D. Careless

Ans: C 14) Mitigate A. Aggravate B. Relieve Ans: A 15) Divergent A. Contrary

C. Eliminate

D. Exhume

B. Coming Together

C. Conversant

D. Controversy

Ans: B 16) Dogmatic A. Skeptical B. Resilient C. Stubborn D. Suspicious Ans: D 17) Clutch A. Hold

B. Grab

C. Release

D. Spread

Ans: C 18) Motley A. Bulky B. Speckled Ans: C 19) Relinquish A. Pursue


Ans: A

C. Homogeneous

D. Different

B. Vanquish

C. Destroy

D. Devastate

20) Transient (I) Permanent


Ans: A

(Ii) Removed

C. ephemeral

D. passing

Directions for Questions 21-26: Read the passage and answer the questions that follow on the basis of the information provided in the passage.
Nature is like business. Business sense dictates that we guard our capital and live from the interest. Nature's capital is the enormous diversity of living things. Without it, we cannot feed ourselves, cure ourselves of illness or provide industry with the raw materials of wealth creation. Professor Edward Wilson, of Harvard University says," The folly our descendants are least likely to forgive us is the ongoing loss of genetic and spices diversity. This will take millions of years to correct". Only 150 plant species have ever been widely cultivated. Yet over 75000 edible plants are known in the wild. In a hungry world, with a population growing by 90 million each year, so much wasted potential in tragic. Medicines from the wild are worth around 40 billion dollars a year. Over 5000 species are known to yield chemical with cancer fighting potential Scientists currently estimate that the total number of species in the world is between 10-30 million with only around 1.4 million identified. The web of life is torn when mankind exploits natural resources in short-sighted ways. The trade in tropical hardwoods can destroy whole forests to extract just a few commercially attractive specimens. Bad agricultural practicetriaggers 24 billion tonnes of top soil erosion a year losting the equivalent of 9 million tonnes of grain output. Cutting this kind of unsuitable exploitation and instituting "sustainable utilisation" will help turn the environmental crisis around. 21. Why does the author compare 'nature' to business ?

A) Because of the capital depletion in nature and business B) Because of the similarity with which one should use both C) Because of the same interest level yield D) Because of the diversity of the various capital inputs. Ans : B 22. "The folly our descendants are least likely to forgive us". What is the business equivalent of the folly the author is referring to ? A) Reducing the profit margin B) Not pumping some money out of profits into the business C) Eroding the capital lease of the business D) Putting interest on capital back into the business Ans: C 23. Which of the following statements in false in context of the given passage ? A) The diversity of plant life is essential for human existence B) Scientists know the usefulness of most plat species C) Chemicals for cancer treatment are available from plants. D) There are around ten times the plant species undiscovered as compared to the discovered ones Ans: B 24.Which of the following correctly reflects the opinion of the author to take care of hunger of the world ? A) Increase the number of the edible plants being cultivates. B) Increase cultivation of the 150 species presently under cultivation C) Increase the cultivation of medical plants D) Increase the potential of the uncultivated edible plants ? Ans: D 25. Which of the following is mentioned as the immediate cause for the destruction of plant species ? A) Soil Erosion Ans: B 26. Choose the word which is nearly same in meaning to the given word as used in the passage,. Wasted A) Consumed Ans: C B) Squandered C) Unutilized D) Unprofitable B) Destruction of habitat C) Cultivation D) Agricultural practices

Directions 27-32 : Pick out the most effective word from the given words to fill in the blank to make the sentence meaningfully complete.
27. Priya is not............. for this kind of a job A) cut in Ans : D 28. He left the book................ the telephone A) around Ans : B 29. The waiter took the plates.................after we had finished eating A) up Ans : C 30. It is fourteen years since I...............him A) saw Ans : A 31. I have done my muddled but...................honest best A) never the less Ans : C 32. It is mainly due to their lethargy that the plan fell................... A) over Ans : C B) out C) through D) off B) rather C) none of these B) have seen C) did see D) had seen B) off C) away D) out B) beside C) besides D) at B) cut up C) cut through D) cut out

Section 2: QUANTITATIVE/LOGICAL REASONING ( 38 questions , 40 minutes )


1. There are 150 weights .Some are 1 kg weights and some are 2 kg weights. The sum of the weights is 260. What is the number of 1kg weights? Ans. 40 2. A is driving on a highway when the police fines him for over speeding and exceeding the limit by 10 km/hr. At the same time B is fined for over speeding by twice the amount by which A exceeded the limit. If he was driving at 35 km/hr what is the speed limit for the road? Ans. 15 kmph 3. A moves 3 kms east from his starting point . He then travels 5 kms north. From that point he moves 8 kms to the east. How far is A from his starting point? Ans. 13 kms 4. A car travels 12 kms with a 4/5th filled tank. How far will the car travel with 1/3 filled tank? Ans. 5 kms

5. The sum of the digits of a two digit number is 8. When 18 is added to the number, the digits are reversed. Find the number? Ans. 35

6. The cost of one pencil, two pens and four pencils, three pens and three erasers cost? Ans. 27

erasers is Rs.22 while the cost of five pencils, four pens and two erasers is Rs.32.How much will three

7. Fathers age is 5 times his son's age. 4 years back the father was 9 times older than son. Find the fathers' present age. Ans. 40 years 8. What number should be added to or subtracted from each term of the ratio17 : 24 so that it becomes equal to 1:2 ? Ans. 10 should be subtracted 9. What is the 12th term of the series 2, 5, 8, .... Ans. 35 10. If 20 men take 15 days to to complete a job, in how many days can 25 men finish that work? Ans. 12 days 11. In a fraction, if 1 is added to both the numerator at the denominator, the fraction becomes 1/2. If numerator is subtracted from the denominator, the fraction becomes 3/4. Find the fraction. Ans. 3/7 12. If Rs.1260 is divided between between A, B and C in the ratio 2:3:4, what is C's share? Ans. Rs. 560 13. A shopkeeper bought a watch for Rs.400 and sold it for Rs.500.What is his profit percentage? Ans. 25% 14. What percent of 60 is 12? Ans. 20% 15. Hansie made the following amounts in seven games of cricket in India: Rs.10, Rs.15, Rs.21, Rs.12, Rs.18, Rs.19 and Rs.17(all figures in crores of course).Find his average earnings. Ans. Rs.16 crore 16. If two pencils cost 8 cents, then how much do 5 pencils cost? Ans. 20 cents 17. Some work is done by two people in 24 minutes. One of them can do this work alone in 40 minutes. How much time does the second person take to do the same work ? Ans. 60 minutes 18. A car is filled with four and half gallons of fuel for a round trip.If the amount of fuel taken while going is 1/4 more than the amount taken for coming, what is the amount of fuel consumed while coming back? Ans.2 gallons

19. The lowest temperature in the night in a city A is 1/3 more than 1/2 the highest during the day. Sum of the lowest temperature and the highest temperature is 100 degrees. Then what is the low temp? Ans.40 degrees 20. Javagal, who decided to go to weekened trip should not exceed 8 hours driving in a day. The average speed of forward journey is 40 miles/hr.Due to traffic on sundays, the return journey's average speed is 30 m/h. How far he can select a picnic spot? Ans. 120 miles 21. A salesperson by mistake multiplied a number and got the answer as 3, instead of dividing the number by 3.What is the answer he should have actually got? Ans. 3 22. A building with height D shadow upto G. What is the height of a neighbouring building with a shadow of C feet. Ans. (C*D)/G 23. A person was fined for exceeding the speed limit by 10 mph. Another person was also fined for exceeding the same speed limit by twice the same. If the second person was travelling at a speed of 35 mph, find the speed limit. Ans. 15 mph 24 A bus started from bus stand at 8.00am, and after staying for 30 minutes at a destination, it returned back to the bus stand. The destination is 27 miles from the bus stand. The speed of the bus is 18mph. During the return journey bus travels with 50% faster speed. At what time does it return to the bus stand? Ans. 11.00am 25. In a mixture, R is 2 parts and S is 1 part. In order to make S to 25% of the mixture, how much of R is to be added? Ans. One part of R 26. Wind flows 160 miles in 330 min, for travelling 80 miles how much time does it require? Ans. 2 hrs 45 mins 27. With a 4/5 full tank a vehicle can travel 12 miles, how far can it travel with a 1/3 full tank Ans. 5 miles 28. There are two trees in a lawn. One grows at a rate 3/5 of the other in 4 years. If the total growth of trees is 8 ft. What is the height of the smaller tree after 2 years Ans. 1 1/2 feet 29. Refer to the figure below. A ship started from P and moves at a speed of I miles per hour and another ship starts from L and moving with H miles per hour simultaneously .Where do the two ships meet? ||---g---||---h---||---i---||---j---||---k---||---l---|| PG H I J K L are the various stops in between denoted by || . The values g, h, i, j, k, l denote the distance between the ports. Ans. Between I and J, closer to J 30. If A is travelling at 72 km per hour on a highway. B is travelling at a speed of 25 meters per second on a highway. What is the difference in their speeds in m/sec. Ans. 1 m/sec

31.

If the word 'ddosszm' is changed to 'central' then what will be the change for 'rtjbl' ? Ans:quick

The word unimpressive was given.they asked us to do change 1st & 2nd,3rd & 4th,so on.then they asked what will be 10th letter from right? Ans: m
32.

The ques on a man,a woman and a boy finish work together in 6 days.man takes 10 days,woman takes 24 days then how much boy will take? Ans:40 days
33. 34.

If DDMUQZM is coded as CENTRAL then RBDJK can be coded as --------Ans. QCEIL

In the word ECONOMETRICS, if the first and second , third and forth ,forth and fifth, fifth and sixth words are interchanged up to the last letter, what would be the tenth letter from right? Ans. word is CENOMOTEIRSC tenth word is R
35. 36.

Find the physical quantity in units from the equation: (Force*Distance)/(Velocity*Velocity) Ans. Ns2/m

37.

Find the value of @@+25-++@16, where @ denotes "square" and + denotes "square root". Ans: 621 If f(0)=1 and f(n)= f(n-1)*n, find the value of f(4). Ans: 24

38.

Section 3: CRITICAL REASONING ( 12 questions , 30 minutes)


Directions 1-12: Answer the questions given below the passage or statement as true, false or can't say. PASSAGE : My father has no brothers. He has three sisters who has two child's each. Answer 1-5 based on the passage 1. My grandfather has two sons . Ans. False 2. Three of my aunts have two sons Ans. Can't say 3. My father is only child to his father

Ans. False 4. I have six cousins from my mother side Ans. Can't say 5. I have one uncle Ans. Can't say (uncle can be from the mother's side as well) PASSAGE: Ether injected into gallbladder to dissolve cholesterol based gallstones. This type one day treatment is enough for gallstones not for calcium stones. This method is alternative to surgery for millions of people who are suffering from this disease.

Answer questions 6-9 based on passage


6.Calcium stones can be cured in one day Ans. False 7. Hundreds of people contains calcium stones Ans. Can't say 8. Surgery is the only treatment to calcium stones Ans. True 9. Ether will be injected into the gallbladder to cure the cholesterol based gall stones Ans. True PASSAGE : Hacking is illegal entry into another computer. This happens mostly because of lack of knowledge of computer networking. With networks one machine can access to another machine. Hacking go about without knowing that each network is accredited to use network facility. Answer questions 10-12 based on passage 10. Hackers never break the code of the company which they work for Ans. Can't say

11. Hacking is the only vulnerability of the computers for the usage of the data Ans. False 12.Hacking is done mostly due to the lack of computer knowledge Ans. False THOSE WHO CLEAR APTITUDE PORTIONS OF THE TEST ARE ELIGIBLE FOR PSYCHOMETRY TEST.

Exactly similar to old pattern and old questions. Total 150 questions Section 4: PSYCHOMETRY (30 MIN)

Psychometry

Direction:
In this section you will find different questions with the same meaning. In all such questions your answer has to be same. for e.g.: In being thrown by chance with a stranger, you wait for the person to introduce himself or herself. (a) Yes (b) No (c) ? It is difficult for you to chat about things in general with people. (a) Yes (b) No (c) ? These two questions have similar meanings. If you answer the first one 'NO' and the second one 'YES', i.e. if you differ in your answers to similar questions you lose marks for every question with the above meaning.

The choices to these questions are: (a) Yes. (b) No. (c) ? 1. 2. 3. 4. 5. 6. 7. 8. 9. 10. 11. 12. 13. 14. 15. 16. 17. 18. 19. 20. 21. 22. 23. 24. 25. 26. 27. 28. 29. 30. 31. 32. 33. 34. 35. 36. 37. 38. 39. 40. 41. 42. 43. 44. 45. 46. 47. 48. 49. 50. 51. 52. 53. 54. 55. 56. 57. 58. 59. 60. 61. 62. 63. 64. 65. 66. You start to work on a project with great deal of enthusiasm. You would rather plan an activity than take part in it. You have more than once taken lead in organizing project or a group of some kind. You like to entertain guests. Your interests change quickly from one thing to another. When you eat a meal with others, you are usually one of the last to finish. You believe in the idea that we should " eat, drink and be merry, for tomorrow we die." When you find that something you have bought is defective, you hesitate to demand an exchange or a refund. You find it easy to find new acquaintances. You are sometimes bubbling over with energy and sometimes very sluggish. You are happiest when you get involved in some projects that calls for rapid action. Other people think of you as being very serious minded. In being thrown by chance with a stranger, you wait for the person to introduce himself or herself. You like to take part in many social activities. You sometimes feel "just miserable" for no good reason at all. You are often so much " on the go" that sooner or later you may wear yourself out. You like parties you attend to be lively. If you hold an opinion that is radically different that expressed by a lecturer, you are likely to tell the person about it either during or after the lecture. It is difficult for you to chat about things in general with people. You give little thought to your failures after they are passed. You often wonder where others get all the excess energy they seem to have. You are inclined to stop to think things over before you act. You avoid arguing over a price with a clerk or sales person. You would dislike very much to work alone in some alone place. You often find it difficult to go to sleep at night because you keep thinking of what happened during the day. You find yourself hurrying to get to places even when there is plenty of time. You like work that requires considerable attention to details. You are satisfied to let some one else take the lead in group activities. You enjoy getting acquainted with people. It takes a lot to get you emotionally stirred up or excited. You work more slowly and deliberately than most people of your sex and age. You are a carefree individual. When people do not play fair you hesitate to say anything about it to them. It bothers you to have people watch you at your work. You have usually been optimistic about your future. You like to have plenty of time to stop and rest. You take life very seriously. You enjoy applying for a job in person. You would like to be a host or hostess for parties at club. You often feel uncomfortable or uneasy. You are the kind of person who is "on the go" all the time. You often crave excitement. The thought of making a speech frightens you. You find it easy to start conversation with strangers. You often feel guilty without a very good reason for it. People think you are a very energetic person. You sometimes make quick decisions that you later wish you hadn't made. You find it difficult to ask people for money or other donations, even for a cause in which you are interested. You are so naturally friendly that people immediately feel at ease with you. You daydream a great deal. You are quick in your actions. You have a habit of starting things and then losing interest in them. When you were a child many of your playmates naturally expected you to be the leader. You sometimes avoid social contacts for fear of doing or saying the wrong thing. You have frequent ups and downs in mood, sometimes with and sometimes without apparent cause. You always seem to have plenty of vigour and vitality. It is difficult for you to understand people who get very concerned about things. When a clerk in a store waits on others who come after you, you call his or her attention to the fact. You would be very unhappy if you were prevented from making numerous social contacts. There are times when your future looks very dark. You sometimes wish that people would slow down a bit and give you a chance to catch up. Many of your friends think you take your work too seriously. You hesitate to walk into a meeting when you know that everyone's eye will be upon you. You limit your friendships mostly to members of your own sex. You almost always feel well and strong. You seem to lack the drive necessary to get as much as other people do.

67. 68. 69. 70. 71. 72. 73. 74. 75. 76. 77. 78. 79. 80. 81. 82. 83. 84. 85. 86. 87. 88.

You make decisions on the spur of the moment. You are rather good at bluffing when you find yourself in difficulty. After being introduced to someone , you just cannot think of things to say to make good conversation. You feel lonesome even when with other people. You are able to work for unusually long hours without feeling tired. You often act on the first thought that comes into your head. At the scene of an accident, you take an active part in helping out. You have difficulty in making new friends. Your mood often changes from happiness to sadness or vice versa without knowing why. You talk more slowly than most people. You like to play practical jokes upon others. You take the lead in putting life into a dull party. You would like to belong to as many clubs and social organizations as possible. There are times when your mind seems to work very slowly and other times when it works very rapidly. You like to do things slowly and deliberately. You are a happy-go-lucky individual. When you are served stale or inferior food in a restaurant, you say nothing about it. You would rather apply for a job by writing a letter than by going through with a personal interview. You are often in low spirits. You are inclined to rush from one activity to another without pausing enough for rest. You are so concerned about the future that you do not get as much fun out of the present as you might. When you are attracted to a person whom you have not met earlier you make an active attempt to get acquainted even though it may be quite difficult. 89. You are inclined to limit your acquaintances to select few 90. you seldom give your past mistakes a second thought. 91. You are less energetic than many people you know. 92. You often stop to analyzed your thoughts and feelings. 93. You speak out in meetings to oppose those whom you feel sure are wrong. 94. You are so shy it bothers you. 95. You are sometimes bothered by having a useless thought come into your mind over and over. 96. You get things in hurry. 97. It is difficult for you to understand how some people can be so unconcerned about the future. 98. You lie to sell things (i.e. to act as a sales person) 99. You are often "Life of the Party". 100. You find daydreaming very enjoyable. 101. At work or at play other people find it hard to keep up with the pace you set. 102. You can listen to a lecture without feeling restless. 103. You would rather work for a good boss than for yourself. 104. You can express yourself more easily in speech than in writing. 105. You keep in fairly uniform spirits. 106. You dislike to be hurried in your work. 107. You sometimes find yourself "crossing bridges before you come to them". 108. You find it somewhat difficult to say "no" to a sales person who tries to sell you something you do not really want. 109. There are only a few friends with whom you can relax and have a good time. 110. You usually keep cheerful in spite of trouble. 111. People sometimes tell you to "slow down" or "take it easy". 112. You are one of those who drink or smoke more than they know they should. 113. When you think you recognize people you see in a public place, you ask them whether you have met them before. 114. You prefer to work alone. 115. Disappointment affect you so little that you seldom think about them twice. 116. You are slow and deliberate in movements. 117. You like wild enthusiasm, sometimes to a point bordering on rowdyism at a football or baseball game. 118. You feel self conscious in the presence of important people. 119. People think of you as being a very social type of person. 120. You have often lost sleep over your worries. 121. You can turn out a large amount of work in a short time. 122. You keep at a task until it is done, even after nearly everyone else has given up. 123. You can think of a good excuse when you need one. 124. Other people say that it is difficult to get to know you well. 125. You daydreams are often about things that can never come true. 126. You often run upstairs taking two steps at a time. 127. You seldom let your responsibility interfere with your having a good time. 128. You like to take on important responsibilities such as organizing a new business. 129. You have hesitated to make or to accept "dates" because of shyness. 130. Your mood is very easily influenced by people around you. 131. Others are often amazed by the amount of work you turn out. 132. You generally feel as though you haven't a care in the world. 133. You find it difficult to get rid of sales person whom you do not care to listen or give your time. 134. You are a listener rather than a talker in a social conversation. 135. You almost always feel that life is very much worth living. 136. It irritates you to have to wait at a crossing for a long freight train to pass. 137. You usually say what you feel like saying at the moment. 138. You like to speak in public. 139. You like to be with people. 140. You generally keep cool and think clearly in exciting situations. 141. Other people regard you as a lively individual. 142. When you get angry, if you let yourself go, you feel better. 143. You seek to avoid all trouble with other people. 144. People seem to enjoy being with you. 145. You sometimes feel listless and tired for no good reason. 146. It is hard to understand why many people are so slow and get so little done. 147. You are fond of betting on horse races and games, whether you can afford it or not. 148. If someone you know has been spreading untrue and bad stories about you, you see the person as soon as possible and have a talk about it. 149. Shyness keep you from being as popular as you should be. 150. You are generally free from worry about possible misfortunes.

Section 1: VERBAL ( 32 Questions ,20 minutes ) Directions for questions 1-10 :Find the synonyms of the following words 1. ACUMEN A. exactness Ans: C 2. DISCRETION A. prudence Ans: A 3. ORDAIN A. arrange Ans: B B. potential C. shrewdness D. bluntness

B. consistency

C. precipice

D. disturbance

B. command

C. contribute

D. establish

4. FLORID A. ornate Ans: A 5. PENITENCE A. liking Ans: C 6. WHET A. stimulate Ans: A 7. LATITUDE A. scope Ans: A 8. dispel A. accumulate Ans: D 9. Orthodox A. heterodox

B. thriving

C. artistic

D. elegant

B. insightful

C. compunction

D. penetrable

b. humorous

c. inculate

d. dampen

B. lamentation

C. globule

D. legislature

B. collect

C. garner

D. deploy

B. unconventional

C. conventional

D. untraditional

Ans: C 10. Alienate A. be friendly Ans: B Directions for questions 11-20:Find the Antonyms of the following words 11 .Dogmatic A. Arbitrary Ans: D 12. Fallible A. Perfect Ans: A 13. Harbinger
A. Forerunner B. herald C. potent D.

B. estrange

C. disarm

D. None of these

B. doctrinal

C. unbending

D. Flexible

B. Imperfect

C. unsound

D. mortal

None of these

Ans: D 14. Intermittent


A. Alternating B. sporadic C. Constant D. None of

these

Ans: C 15. Pensive A. ignorant Ans: A 16. Concur


A. harmonize B. coincide C. assent

B. Brooding

C. pondering

D. meditative

D. Conflict

Ans: D

17.Furtive A. Open Ans: B 18. Efface A. Obliterate Ans: B 19. Pretentious A. Ostentatious Ans: C 20. Veer A. avert Ans: D Directions for Questions 21-26: Read the passage and answer the questions that follow on the basis of the information provided in the passage. Amnesty International's charge that 'tens of thousands' of political prisoners, including prisoners of conscience, are languishing in Indian jails and that prisoners are routinely tortured in this country has to be seen in a much wider context than the organisation's annual report cares to do. In its overall appraisal of 151 countries, Amnesty has accused 112 of torturing prisoners, 63 of harboring prisoners of conscience, 61 of resorting to political killings and 53 of detaining people without a trial. Of these apparentlyoverlapping categories, India seems to have been excluded from the list of the 61 which undertake political killings. The report has however, pointed out that scores of people in India die of torture in police and military custody and that many also simply disappear. Clearly, only a thin line separates the 61 charged with political murder from the rest. Before coming to such conclusions, however, it may also be necessary to classify the various countries according to their political systems. Torture by the security forces and killings at the behest of the government make no difference to the victims whether they are in a democratic country or a totalitarian one. It is also nobody's case that a democratic country is less culpable than a dictatorship in the event of human rights violations. But the point perhaps still needs to be made that torture or 'disappearances' represent a failure of the system in a democracy in contrast to being an integral part of state policy in a country ruled by an autocrat who is answerable to no one. B. skew C. whirl Dstay B. pompous C. Down-to-earth D. conceited B. Engrave C. eradicate D. None of these B. Surreptitious C. clandestine D. None of these

India may be guilty of keeping 'tens of thousands' behind bars and of the other human rights abuses mentioned by qualitatively different place from a totalitarian country. It is in this respect that Amnesty has been lass than fair. It has chosen to ignore the distinctions between the good, the bad and the ugly. The openness of Indian society will be evident to anyone who spends half an hour in one of its chaotic market-places or visits the law courts or watches a political rally or reads a newspaper or strikes up a conversation with any person on the roads. There is no sense of fear in India, as in a dictatorship. There is also scope for securing relief from the heavyhanded behaviour of the authorities, even if the human rights commission has not yet lived up to expectations. Unless such points are recognised, Amnesty's assessment will seem to be a dry recital of statistics which may pillory India simply because of its larger population. Mercifully, Amnesty nowadays at least notes that the terrorists also indulge in human rights violations and that India has to cope with several insurgencies fomented by a country where the military does not always seem to be under the control of the elected government. True, there is much that is the way the terrorist challenge is activating the self-correcting mechanism within a democracy and not merely on painting a grim, even biased picture. 21. In the report, India has been excluded from which of the following categories of violating human rights? A) Torturing prisoners B) Detaining without trial C) Political killings D) harbouring prisoners of conscience E) None of these Ans: C 22. Which of the following is not true in the context of the passage ? A) India is guilty of some human rights abuses B) Amnesty International appraised all the democratic countries C) There is overlapping of cases in the categories of human right abuses D) India was one of the countries appraised by Amnesty International E) The report notes that the terrorists also violate human rights Ans: B

23. According to the passage, through which media of forum Amnesty International has hurled the charges? A) Seminar on Human rights B) Its Regional Meet C) Its annual Report D) Its International Meet E) None of these Ans: C 24. The author of the passage A) agrees with the report B) disagrees with the report C) disagrees that conditions of prisons in India is bad D) supports the totalitarian approach E) disagrees with report on terrorists Ans: B 25. The Amnesty Internationals report is based on the information of how many countries ? A) 63 B) 112 C) 131 Ans: E 26. The author suggests classification of various countries on the additional dimension. Which of the following is that dimension ? A) Economic progress D) Political systems Ans: B Directions 27-32 : Pick out the most effective word from the given words to fill in the blank to make the sentence meaningfully complete. B) Human rights E) None of these C) Industrial Progress D) 115 E) None of these

27. The boy you met yesterday is in class.................. A) ninth Ans : C 28. There was some confusion.................the agreement A) on B) in Ans : D 29. Speak loudly as he is slow.........................hearing A) in Ans : C 30. Will you..................may dog while I am on tour ? A) look out Ans : D 31. Savitha is disgusted...................the habits of her husband A) of B) from C) with D) at B) look up C) look on D) look after B) about C) at D) of C) around D) over B) the ninth C) nine D) the nine

Ans : C 32. Every man craves....................recognition A) for Ans : D Section 2: QUANTITATIVE/LOGICAL REASONING ( 38 questions , 40 minutes ) 1. If a + b + c = 0, then (a3 + b3 + c3) abc is equal to: (A) 1 Ans : C (B) 2 (C) 3 (D) 9 B) about C) at D) after

2.The difference between compound interest and simple interest for 3 years at 5% per annum can be found out by multiplying the principal by: (A) 1.7625 Ans : A 3.The simple interest on a sum of money is 1/9 th of the sum and the number of years and the rate per cent per annum are equal. The rate per cent per annum is: (A) 3 1/3 Ans : A 4.A man invested Rs 5,000 at some rate of simple interest and Rs 4,000 at 1% higher rate of interest. If the interest in both the cases after 4 years is same the rate of interest in the former case is: (A) 4% Ans : A 5. A sum of money at simple interest rate amounts to Rs 4,025 in 3 years and to Rs 4,550 in 6 years at the same rate of interest. Find the sum and the rate of interest per annum. (A) Rs 2,500, 6% Ans : C 6. On a certain map of India the actual distance of 1450 kms between two cities Delhi and Kolkata is shown as 5 cms. What scale is used to draw the map? (A) 1 : 15 106 Ans : D 7. The ratio between the third proportional of 12 and 30 and mean proportional of 9 and 25 is: (A) 2 : 1 Ans : B (B) 5 : 1 (C) 7 : 15 (D) 9 : 14 (B) 1 : 20 106 (C) 1 : 25 106 (D) 1 : 29 106 (B) Rs 3,000, 5% (C) Rs 3,500, 5% (D) Rs 4,500, 4% (B) 5% (C) 6 1/4% (D) 8 1/3 % (B) 5 (C) 6 2/3 (D) 10 (B) 0.7625 (C) 0.07625 (D) 0.007625

8. Rs 1,050 are divided among P, Q and R. The share of P is 2/5 of the combined share of Q and R. P gets: (A) Rs 320 Ans : B ma + nc 9. If a : b = c : d, then ________ mb + nd (A) an : mb Ans : C 10. How many even numbers of four-digits can be formed with digits 1, 2, 3, 4, 5, 6 (repetition of the digit is allowed)? (A) 648 Ans : A 11. There is a number lock with four rings. How many attempts at the maximum would have to be made before getting the right number? (A) 104 Ans : C 12. There are four letters and four envelops addressed to different persons. In how many ways can wrong choices be made? (A) 64 Ans : D 13. There are 10 points on a straight line AB and 8 points on another AC, none of them being A. How many triangles can be formed with these points as vertices? (A) 720 (B) 640 (C) 816 (D) 680 (B) 23 (C) 16 (D) 255 (B) 255 (C) 104 1 (D) 256 (B) 180 (C) 1296 (D) 540 (B) m : n (C) a : b (D) dm : cn is equal to: (B) Rs 300 (C) Rs 200 (D) Rs 420

Ans : B 14. A circular ground of circumference of 88m. A strip of land 3m wide inside and along the circumference is to be leveled. What is the expenditure if leveling cost is Rs. 7 per metre square. Ans. 1650 15. There are 4 boys and 3 girls. What is the probability the boys and girls sit alternately? Ans 1/35 16. Two trains are 2 kms apart. Speed of one train is 20m/s and the other train is running at 30 m/s . Lengths of the trains are 200 and 300m. In how much time do the trains cross each other? Ans. 50 seconds 17. A& B are two players. They need to select one number from 1 to 25. If both the players select the same numbers they will win the prize. What is the probability of not winning in a single trial? Ans 24/25 18. Four different integers are in increasing AP such that one number is equal to sum of the squares of the other three numbers. Ans 1,0,1,2 19. In a company there were 75 % skilled employees and the remaining unskilled. 80% of the skilled and 20% of the unskilled were permanent. If the temporary employees were 126 find the total number of employees. Ans 360 20. A person sells a horse at 12.5% loss. If he sells for 92.5 more, he will have a profit of 6%. What is the CP? Ans 500 21. One tap takes 15 min to fill, another tap takes 12 min to fill and the third tap can empty in 20 min. In how much time the tank would be full.

Ans 10 min 22. Two trains are separated by 200km. One leaves at 6:00 am from Delhi and reaches Merrut at 10:00 am. Another train leaves from Merrut at 8:00 am and reaches Delhi at 11:30 am. At what time two trains meet each other? Ans 8:56 am 23. If the height of the triangle decreases by 40% and the breadth increases by 40%. Then the effect on the area is Ans. 16% decrease. 24. A grandfather has 5 sons and daughters and 8 grandchildren.. They have to be arranged in arrow such that the first 4 seats and last four seats are to be taken by grandchildren and the grandfather would not sit adjacent to any of the grandchildren. Ans. 4 * 8! * 5! 25. A farmer has a rectangular plot. He wants to do fencing along one of the side with the help of the posts. Two posts being on two corners. He brings 5 post less than what he has initially plan because of which the distance between two consecutive post became 8 m instead of 6 m.. What is the length of the side and no of post? Ans. 120 , 16 26. A circular ground of circumference of 88m. A strip of land 3m wide inside and along the circumference is to be leveled. What is the expenditure if leveling cost is Rs. 7 per metre square ? Ans. 1650 27. Four horses are tethered at the four corners of a square of side 14cm such that two horses along the same side can just reach each other. They were able to graze the area in 11 days. How many days will they take in order to graze the left out area? Ans. 3

28. Six bells commence tolling together and toll at intervals 2,4,6,8,10 and 12 seconds respectively. In 30 minutes how many times they toll together. a) 4 b) 10 c) 15 d) 16

Ans: d) 29. The number of coins 1.5 cm in diameter and 0.2cm thick to be melted to form a right circular cylinder of height 10 cm and diameter 4.5 cm is: a) 380 Ans: b) 30. The size of a wooden block is 5 * 10 * 20 cm3. How many whole such blocks you will take to construct a solid wooden cube of minimum size? a) 6 Ans: b) 31. Find the odd man out - 1050, 510, 242, 106, 46, 16, 3 a) 510 b) 242 c) 106 d) 46 b) 8 c) 12 d) 16 b) 450 c) 472 d) 540

Ans: C) 32. A clock loses 10 minutes each hour. If the clock is set correctly at noon, what time is it when it reads 3 PM? Ans: 8 minutes. 33. If 8 crows can steal 8 buttons in 8 minutes, how long will it take 16 crows to steal 16 buttons? 32 buttons 34. The amount of water in a tank doubles every minute. The tank is full in an hour. When was the tank half full? Ans: 59 minutes.

35. A laborer can dig a hole 8 feet square and 8 feet deep in 8 days. How long will it take him to dig a hole 4 feet square and 4 feet deep? Ans: 1 day 36. A clock loses 10 minutes each hour. If the clock is set correctly at noon, what time is it when it reads 3 PM? Ans: 3:36 PM. 37. If 2 miles of fence enclose a square plot of 160 acres, how large a square will 4 miles of fence enclose? Ans: 640 acres or 1 square mile. 38. A woman travels 1 mile south, then one mile west, and then mile north, and arrives at her starting point. Where is she? Ans: North Section 3: CRITICAL REASONING ( 12 questions , 30 minutes)
Directions1-12: Answer the questions given below the passage or statement as true, false or can't say.

PASSAGE : Copernicus is the intelligent. In the days of Copernicus the transport and technology development was less & it took place weeks to communicate a message at that time, wherein we can send it through satellite with in no time. Even with this fast developments it has become difficult to understand each other. Answer questions 1-4 based on passage . 1. People were not intelligent during Copernicus days Ans. False 2. Transport facilities are very much improved in noe a days Ans. Can't say 3. Even with the fast developments of the technology we can't live happily. Ans. Can't say 4. We can understand the people very much with the development of communication

Ans. False. 3. Critical Reasoning ( 12 questions , 30 minutes) PASSAGE: Senior managers warned the workers that because of the introductory of Japanese industry in the car market. There is the threat to the workers. They also said that there will be the reduction in the purchase of the sales of car in public. the interest rates of the car will be increased with the loss in demand. Answer questions 5-8 based on passage . 5. Japanese workers are taking over the jobs of Indian industry. Ans. False 6.Managers said car interests will go down after seeing the raise in interest rates. Ans. True 7. Japanese investments are ceasing to end in the car industry. Ans. False 8. People are very interested to buy the cars. Ans. False PASSAGE: There should be copyright for all arts. The reels has came that all the arts has come under one copy right society, they were use the money that come from the arts for the developments . There may be a lot of money will come from the Tagore works. We have to ask the benifiters from Tagore work to help for the development of his works. Answer questions 35-39 based on passage . 9. Tagore works are came under this copy right rule. Ans. False 10. People are free to go to the public because of the copy right rule. Ans. Can't say

11. People gives to theater and collect the money for development. Ans. Can't say

12. We have ask the Tagore residents to help for the developments of art. Ans.Can't say Section 1: VERBAL ( 32 Questions ,20 minutes ) Directions for questions 1-10:Find the synonyms of the following words 1. POTENTIAL a. latent Ans: a 2. EXTRICATE a. terminate Ans: c 3. DISPARITY a. inequality Ans: a 4. TO CONFISCATE a. to harass Ans: d

b. hysterical

c. conventional

d. symmetrical

b. isolate

c. liberate

d. simplify

b. impartiality

c. unfairness

d. twist

b. to repulse

c. to console

d. to appropriate

5. PIOUS a. historic Ans: b 6. CARGO a. cabbage Ans: d 7. OVATION a. oration Ans: d 8. Candid

b. devout

c. multiple

d. fortunate

b. camel

c. lance

d. freight

b. gesture

c. emulation

d. applause

A. intriguing Ans: C 9. Murky A. overcast Ans: A 10. Guile A. Openness Ans: B

B. sly

C. frank

D. cunning

B. dazzling

C. Clear

D. fulgurant

B. slyness

C. fair-dealing

D. ingenuousness

Directions for questions 11-20:Find the Antonyms of the following words 11. Stern A. Lenient Ans: A 12.spry A. agile Ans: D 14. spurn A. weave Ans: D 15. slothful A. summit Ans: D 16. sluggish A. wasteful B. brisk C. baseball D. chilly B. animal C. lazy D. industrious B. turn C. sew D. embrace B. young C. adult D. doddering B. Crabby C. Tenant D. Unreasonable

Ans: B 17.scrawny A. bane Ans: C 18. tolerance A. intolerance Ans: A 19. susceptible A. sensitive Ans: D 20. stingy A. miserly Ans: B Directions for Questions 21-26: Read the passage and answer the questions that follow on the basis of the information provided in the passage. Recent advances in science and technology have made it possible for geneticists to to find out abnormalities in the unborn foetus and take remedial action to rectify some defects which would otherwise prove to be fatal to the child. Though genetic engineering is still at its infancy, scientists can now predict with greater accuracy a genetic disorder. It is not yet in a position to predict when exactly a genetic disorder will set in. While they have not yet been able to change the genetic order of the gene in germs, they are optimistic and are holding out that in the near future they might be successful in achieving this feat. They have, however, acquired the ability in manipulating tissue cells. However, genetic mis-information can sometimes be damaging for it may adversely affect people psychologically. Genetic information may lead to a tendency to brand some people as inferiors. Genetic information can therefore be abused and its application in deciding the sex of the foetus and its subsequent abortion is now hotly debated on ethical lines. But on this issue geneticists cannot be squarely blamed though this charge has often been levelled at them. It is mainly a societal problem. At present genetic engineering is a costly process of detecting disorders but scientists hope to reduce the costs when technology becomes more advanced. This is why much progress in this area has been possible in scientifically advanced and rich countries like the U.S.A, U.K and Japan. It remains to be seen if in the future this science will lead to the development of a race of supermen or will be able to obliterate disease from this world. B. swayable C. amenable D. impervious B. forbearance C. lenience D. acceptance B. skinny C. obese D. lean

B. generous

C. parsimonious

D. sparing

21. Which of the following is the same in meaning as the phrase 'holding out' as used in the passage A) catching Ans: B 22. According to the passage, the question of abortion is A) ignored B) hotly debated already settled Ans: B 23. Which of the following is the same in meaning as the word 'obliterate' as used in the passage. A) wipe off B) eradicate Ans: B 24. Which of the following is the opposite in meaning to the word 'charged' as used in the passage A) calm Ans: D 25. Which of the following is not true of the genetic engineering movement. A) Possibility of abuse B) It is confronted by ethical problems C) Increased tendency to manipulate gene cells D) acquired ability to detect genetic disorders in unborn babies. E) acquired ability to manipulate tissue cells Ans: C 26. Which of the following is the same in meaning as the word 'feat' as used in the passage A) process Ans: E B) focus C) fact D) possibility E) goal B) disturbed C) discharged D) settled E) peaceful C) give birth to D) wipe out E) very literate C) unanswered D) left to the scientists to decide E) B) expounding C) sustaining D) restraining E) controlling

Directions 27-32 : Pick out the most effective word from the given words to fill in the blank to make the sentence meaningfully complete. 27. He preferred cricket.....................any other sport. A) against Ans: B 28. There is a disturbing nexus.....................politics and sports in India A) between Ans: A 29.Don't loiter.................the street A) in Ans: B 30. The terms given to you are not acceptable ..............me A) for Ans: D 31. You have played a great role, for ..................your help I possibly would have landed myself into a problem. A) after B) despite Ans: E 32. Fate smiled..............him in all his ventures A) upon B) on C) at Ans: B Section 2: QUANTITATIVE/LOGICAL REASONING ( 38 questions , 40 minutes ) 1. A pupil's marks were wrongly entered as 83 instead of 63. Due to that the average marks for the class got increased by half. The number of pupils in the class is D) over C) unless D) although E) without B) by C) with D) to B) about C) on D) into B) with C) among D) by B) to C) over D) than

A) 10 B) 20 C) 40 Ans: C

D) 73

2. The average weight of A, B and C is 45 kg. If the average weight of A and B be 40 kg and that of B and C be 43 kg, then the weight of B is A) 17 kg B) 20 kg Ans: D 3. The average monthly salary of 20 employees in an organisation is Rs. 1500. If the manager's salary is added, then the average salary increases by Rs. 100. What is the manager's salary? A) Rs.2000 Ans: C 4. The average of five consecutive numbers in n. If the next two numbers are also included, the average will : A) remain the same B) increase by 1 Ans: B 5. In an election between two candidates, one got 55% of the total valid votes, 20% of the votes were invalid. If the total number of votes was 7500, the number of valid votes that the other candidate got, was A) 2700 B) 2900 Ans: A 6. Two tailors X and Y are paid a total of Rs. 550 pe week by their employer. If X is paid 120 percent of the sum paid to Y, how much is Y paid per week A) Rs. 200 Ans : B B) Rs. 250 C) Rs.300 C) Rs. None of these C) 3000 D) 3100 C) increase by 1.4 D) increase by 2 R) Rs.2400 C) Rs. 3600 D) Rs.4800 C) 26 kg D) 31 kg

7. A student secures 90%, 60% and 54% marks in test papers with 100, 150 and 200 respectively as maximum marks. The percentage of his aggregate is A) 64 B) 68 C) 70 Ans: A 8. A man gains 20% by selling an article for a certain price. If he sells it at double the price, the percentage of profit will be A) 40 B) 100 C) 120 Ans: D 9. Profit earned by selling an article for Ra. 1060 is 20% more than the loss incurred by selling the article for Rs. 950. At what price should be article be sold to earn 20% profit.? A) 980 B) 1080 Ans: D 10. The ages of A and B are in the ration of 3:1. Fifteen years hence, the ratio will be 2:1 Their present ages are A) 30 years, 10 years Ans: B 11. The electricity bill of a certain establishment is partly fixed and partly varies as the number of units of electricity consumed. When in a certain month 540 units are consumed, the bill is Rs. 1800. In another month 620 units are consumed and the bill is Rs. 2040. In yet another month 500 units are consumed. The bill for that month would be A) Rs.1560 Ans: B 12. A and B are two alloys of gold and copper prepared by mixing metals in the ratio 7: 2 and 7: 11 respectively. B) Rs. 1680 C) Rs. 1840 D) Rs. 1950 B) 45 years, 15 years C) 21 years, 7 years D) 60 years, 20 years C) 1800 D) None of these D) 140 D) None of these

If equal quantities of the alloys are melted to form a third alloy C, the ratio of gold and copper in C will be A) 5:7 Ans: C 13. Three men, four women and six children can complete a work in seven days. A women does double the work a man does and a child does half the work a man does. How many women alone can complete this work in 7 days A) 7 B) 8 C) 12 Ans: A 14. A man, a women and a boy can complete a job in 3, 4 and 12 days respectively. How many boys must assist 1 man and 1 women to complete the job in 1/4 of a day. A) 1 B) 4 C) 19 Ans: D 15. If 6 men and 8 boys can do a piece of work in 10 days while 26 men and 48 boys can do the same in 12 days, the time taken by 15 men and 20 boys in doing the same type of work will be : A) 4 days Ans: A 16. With a uniform speed a car covers the distance in 8 hours. Had the speed been increased by 4 km/hr, the same distance could have been covered in 7/1/2 hours. What is the distance covered? A) 420 km Ans: B 17. In a flight of 600 km, a aircraft was slowed down due to bad weather. Its average speed for the trip was reduced by 200 km/ hr and the time of flight increased by 30 minutes. The duration of the flight is : B) 480 km C) 640 km D) Cannot determined E) None of these B) 5 days C) 6 days D) 7 days D) 41 D) Cannot be determined E) None of these B) 5:9 C) 7:5 D) 9:5

A) 1 hour Ans: A

B) 2 hours

C) 3 hours

D) 4 hours

18.It takes eight hours for a 600 km journey, if 120 km is done by train and the rest by car. It takes 20 minutes more, if 200 km is done by trai and the rest by car. The ratio of the speed of the train to that of the car is : A) 2 : 3 Ans: C 19. A goods train runs at the speed of 72 kmph and crosses a 250 m long platform in 26 seconds. What is the length of the goods train ? A) 230 m Ans: D 20. A 300 metre long train crosses platform in 39 seconds while it crosses a signal pole in 18 seconds. What is the length of the platform ? A) 320 m Ans: B 21. A 270 metres long train running at the speed of 120 kmph crosses another train running in opposite direction at the speed of 80 kmph in 9 seconds. What is the length of the other train A) 230 m Ans: A 22. A man can row at 5 kmph in still water. If the velocity of current is 1 kmph and it takes him 1 hour to row to a place and come back, how far is the place ? A) 2.4 km Ans: A B) 2.5 km C) 3 km D) 3.6 km B) 240 m C) 260 m D) 320 m E) None of these B) 350 m C) 650 m D) Data inadequate E) None of these B) 240 m C) 260 m D) 270 m B) 3 : 2 C) 3 : 4 D) 4 : 3

23. A boat takes 19 hours for travelling downstream from point A to point B and coming back to a point C midway between A and B. If the velocity of the stream is 4 kmph and the speed of the boat in still water is 14 kmph, what is the distance between A and B ? A) 160 km Ans : B 24. Peter invested an amount of Rs. 12000 at the rate of 10 p.c.p.a. simple interest and another amount at the rate of 20 p.c.p.a simple interest. The total interest earned at the end of one year on the total amount invested become 14 p.c.p.a. Find the total amount invested. A) 20,000 B) 22,000 Ans: A 25. The rates of simple interest in two banks A and B are in the ratio 5 : 4. A person wants to deposit his total savings in two banks in such a way that he received equal half-yearly interest from both. he should deposit the savings in banks A and B in the ration : A) 2 : 5 Ans : B 26. What will be the cost of gardening 1 metre broad boundary around a rectangular plot having perimeter of 340 metres at the rate of Rs.10 per square metre ? A) Rs. 1700 Ans : C 27.The length and breadth of the floor of the room are 20 feet and 10 feet respectively. Square tiles of 2 feet length of different colours are to be laid on the floor. Black tiles are laid in the first row on all sides. If white tiles are laid in the one-third of the remaining and blue tiles in the rest, how many blue tiles will be there ? A) 16 B) 24 C) 32 D) 48 E) one of these B) Rs. 3400 C) Rs. 3440 D) Cannot be determined E) None of these B) 4 : 5 C) 5 : 2 D) 5 : 4 C) 24,000 D) 25,000 E) None of these B) 180 km C) 200 km D) 220 km

Ans: A 28. A park square in shape has a 3 metre wide road inside it running along its sides. To area occupied by the road is 1764 square metres. What is the perimeter along to outer edge of the road ? A) 576 metres Ans: B 29. At what time between 7 and 8 o'clock will the hands of a clock be in the same straight line but not together A) 5 min. past 7 Ans: D 30. A watch which gains uniformly is 2 minutes low at noon on Monday and is 4 min. 48 sec fast at 2 p.m on the following Monday. When was it correct ? A) 2 p.m on Tuesday B) 2 p.m on Wednesday C) 3 p.m on Thursday D) 1 p.m on Friday Ans: B 31. How many shares of market value Rs. 25 each can be purchased for Rs. 12750, brockerage being 2 % ? A) 450 B) 500 C) 550 Ans : B 32. A Man buys Rs. 50 shares in a company which pays 10% dividend. If the man gets 12.5% on his investment, at what price did he buy the shares ? A) Rs. 37.50 Ans: B 33. In how many different ways can be letters of the word, 'RUMOUR' be arranged ? A) 180 B) 90 C) 30 Ans : A D) 720 E) None of these B) Rs. 40 C) Rs. 48 D) Rs. 52 D) 600 B) 5 2/11 min. past 7 C) 5 3/11 min. past 7 D) 5 5/11 min. past 7 B) 600 metres C) 640 metres D) Data inadequate E) None of these

34. In how many different ways can the letters of the word 'AUCTION' be arranged in such a way that the vowels always come together ? A) 30 B) 48 C) 144 Ans : D 35. In a group of 6 boys and 4 girls, four children are to be selected. In how many different ways can they be selected such that at least one boy should be there ? A) 159 B) 194 C) 205 Ans: D 36. A box contains 5 green, 4 yellow and 3 white marbles. Three marbles are drawn at randum. What is the probability that they are not of the same colour ? A) 3/44 Ans : D 37. In a box, there are 8 red, 7 blue and 6 green balls. One ball is picked up randomly. what is the probability that it is neither red nor green ? A) 2/3 Ans : D 38. The true discount on Rs. 1760 due after a certain time at 12% per annum is Rs. 160. The time after which it is due is A) 6 months Ans : D Section 3: CRITICAL REASONING ( 12 questions , 30 minutes)
Directions 1-12 : Answer the questions given below the passage or statement as true, false or can't say.

D) 576

E) none of these

D) 209

E) None of these

B) 3/55 C) 52/55

D) 41/44

B) 3/4 C) 7/19

D) 8/21

E) 9/21

B) 8 months

C) 9 months

D) 10 months

PASSAGE :Glaciers begin to form where snow remains year-round and enough of it accumulates to transform into ice. New layers of snow compress the previous layers and this compression forces the icy snow to re-crystallize, forming grains similar in size and shape to

cane sugar. Gradually the grains grow larger and the air pockets between the grains get smaller, meaning that the snow slowly becomes more dense. After about two winters, the snow turns into firn, an intermediate state between snow and ice. Over time the larger ice crystals become more compressed and even denser, this is known as glacial ice. Glacial ice, because of its density and ice crystals, often takes a bluish or even green hue. Answer questions 1- 5 based on passage . 1. Glaciers cannot form where snow does not remain all year round. A. True Ans: A 2. Firn is less dense than snow but more dense than ice A. True Ans: B 3. Glacial ice is always greenish or bluish in color. A. True Ans: B 4. Snow falls every year in areas where glaciers form. A. True Ans: C 5. The increase in density is caused by the grains becoming smaller. A. True Ans: B PASSAGE :A power of attorney or letter of attorney in common law systems or mandate in civil law systems is an authorization to act on someone else's behalf in a legal or business matter. The person authorizing the other to act is the "principal" or "grantor", and the one authorized to act is the "agent" or "attorney-in-fact". The attorney-in-fact acts "in the principal's name," signing the principal's name to documents and filing suit with the principal's name as plaintiff, for example. As one kind of agent, an attorney-in-fact is a fiduciary for the principal, so the law requires an attorney-in-fact to be completely honest with and loyal to the principal in their dealings with B. False C. Can't say B. False C. Can't say B. False C. Can't say B. False C. Can't say B. False C. Can't say

each other. If the attorney-in-fact is being paid to act for the principal, the contract is a separate matter from the power of attorney itself, so if that contract is in writing, it is a separate document, kept private between them, whereas the power of attorney is intended to be shown to various other people. The power of attorney may be oral, such as asking someone else to sign your name on a cheque because your arm is broken, or may be in writing. Many institutions, such as hospitals, banks, and the I.R.S., require a power of attorney to be in writing before they will honor it, and they usually want to keep an original for their records. Answer questions 6-10 based on passage . 6.The agent grants the principal the power to act on behalf of the grantor. A. True Ans: B 7.All contracts between the principal and the agent must be made public. A. True Ans: B 8. The power of attorney may be granted verbally. A. True Ans: A 9. Only a legal professional can be granted the power of attorney. A. True Ans: C 10. In civil law systems the power of attorney is referred to as a mandate. A. True Ans: A PASSAGE :Any one who has has systematic exam phases will have perceived a profound although not a prolif of asymmetry whether or not the exception is volitions and self control of spontaneous appeal to predict facial as symmetry as does the type of emotion portrayed. position can not displace symmetric at left side regret of a negative emotion is more common posed B. False C. Can't say B. False C. Can't say B. False C. Can't say B. False C. Can't say B. False C. Can't say

expression negation emotions are likely to be symmetric representation and where as symmetric occurs relative left sided expression is more common. Answer questions 11-12 based on passage . 11. Any angry person is more likely to have left sided expression than some one who has smiling A. True Ans: A 12. An actor is likely to smile symmetric when acting A. True Ans: B B. False C. Can't say B. False C. Can't say

We had an online test on 2nd Jan. The instructions will be given by the TCS officials. Listen very carefully to the instructions. A silly mistake will prove very costly. 1. Open the internet explorer and type the address, given by TCS official, in the address bar. 2. The site opens and asks you login id and password. Those will also be provided to you. 3. Once you login, you might get a page Sample Test. Ignore it and click Go to main Test on that same page. Now your test starts. 4. The test has 3 sections of 20 mins, 30 mins and 40 mins. Test starts with section 1. 5. On the right top corner you will have a timer counting down and links to ques of that section. From the timer you can see the time left for that section and through the ques links you can move to any ques within that section only. 6. The ques links have 3 color codes- Red Not Visited Orange- Visited But not attempted Green- Attempted 7. There will be two buttons at the bottom of the page- Exit and Submit 8. After answering a ques click the Submit button to register your answer. You can always come back and change it within the stipulated time. 9. EXIT button takes you out of the section. So click it ONLY after you have attempted all ques of a section. 10. After you finish all 3 sections CLICK EXIT. DONT close the main window WITHOUT clicking EXIT! Online TestThe test is divided into 3 sections. There is NO negative marking and NO sectional cut off. VERBAL SECTION: ( antonyms ,synonyms n reading comprehension) : 10 antonyms were there and 10 synonyms were there , all from BARRON'S GRE 12TH EDITION exercise n model test paper with same options.

2 reading comprehension was there . Mainly they are para completion. APTITUDE SECTION : 1)My flight takes of at 2am from a place at 18N 10E and landed 10 Hrs later at a place with coordinates 36N70W. What is the local time when my plane landed? 6:00 am b) 6:40am c) 7:40 d) 7:00 e) 8:00 Sol) The destination place is 80 degree west to the starting place. Hence the time difference between these two places is 5 hour 20 min. (=24hr*80/360). When the flight landed, the time at the starting place is 12 noon (2 AM + 10 hours). Hence, the time at the destination place is 12 noon - 5:20 hours = 6: 40 AM 2) In a two-dimensional array, X (9, 7), with each element occupying 4 bytes of memory, with the address of the first element X (1, 1) is 3000, find the address of X (8, 5) Sol) initial x (1,1) = 3000 u have to find from x(8,1)so u have x(1,1),x(1,2) ... x(7,7) = so u have totally 7 * 7 = 49 elements you need to find for x(8,5) ? here we have 5 elements each element have 4 bytes : (49 + 5 -1) * 4 = 212 -----( -1 is to deduct the 1 element ) 3000 + 212 = 3212 3) what's the answer for that : A, B and C are 8 bit no's. They are as follows: A -> 1 1 0 0 0 1 0 1 B -> 0 0 1 1 0 0 1 1 C -> 0 0 1 1 1 0 1 0 ( - =minus, u=union) Find ((A - C) u B) =? Sol)To find A-C, We will find 2's compliment of C and them add it with A, That will give us (AC) 2's compliment of C=1's compliment of C+1 =11000101+1=11000110 AC=11000101+11000110 =10001001 Now (A-C) U B is .OR. logic operation on (A-C) and B 10001001 .OR . 00110011 The answer is = 10111011, Whose decimal equivalent is 187. 4) Which one among have higher deviation:1. 5,0,-5, 5,0,-5 2. -5,-5,-5,-5,5,-5 3. 5,-5,5,-5,5,-5 4. 5,5,5,5,5,5 Sol) 3. 5,-5,5,-5,5,-5 whatever number they give you the ans must be either in (+,-,+,-) or in (,+,-,+)form 5)Find the physical quantity represented by MOMENTUM *VELOCITY] / [LENGTH * ACCELERATION]? Sol) mass 6)Find the result of the following _expression if, M denotes modulus operation, R denotes round-off, T denotes truncation: M(373,5)+R(3.4)+T(7.7)+R(5.8) Sol) 373/5=rem(3) 3+3+7+6=19.ans 7)Which of the following are orthogonal pairs? a. 3i+2 b. i+1 c. 3-2i d. -7i+1 Sol)(3i+2)(+i)=2i+3 (3i+2)(-i)= -2i+3

ANS: (A)& (C). 8)Find d Odd one out? ( I m giving u al as dere was 3 ques on dat) a. LINUX b. WINDOWS 98 c. SOLARIS d. SMTP b. JAVA b. LISP c. Small talk d. Eiffel c. HTTP b. ARP c. SMTP d. SAP d. Linux b.windows NT c. SQL server d. UNIX e. SAP b. ARP c. WAP d.TCP IP f. Oracle b. Linux c. Ingress d. DB2 g. SMTP b. WAP c. SAP d. ARP h. WAP b. HTTP c. BAAN d. ARP i. SQL b. DB2 c. SYBASE d. HTTP 9)The size of a program is N. And the memory occupied by the program is given by M = square root of 100N. If the size of the program is increased by 1% then how much memory now occupied ? Sol) M=sqrt(100N) N is increased by 1% therefore new value of N=N + (N/100) =101N/100 M=sqrt(100 * (101N/100) ) Hence, we get M=sqrt(101 * N) 10)A power unit is there by the bank of the river of 750 meters width. A cable is made from power unit to power plant opposite to that of the river and 1500mts away from the power unit.The cost of the cable below water is Rs.15/- per meter and cost of cable on the bank is Rs.12/-per meter. Find the total of laying the cable. sol)750*15+(1500-750)*12=20250 11) Which shape will be obtained by using these values of X & Y? X Y 0 0.00001 10 1.02 100 1.72 1000 3.00 9999 4.72 Sol): Y= log10(X) 12)Find d singularity matrix from a given set of matrices? a) 2 3 b) 3 9 c) 0 5 d) 9 8 45 13 15 45 Sol): b (Hint: det (A) = 0) 13)In d given series {b v f q q m b v f m g v f f v f m b v f b g d v f a m v f v f f ...) How many vs are dere such that each v is followed by f next to it, if the f is not followed by q next to it? Sol): try it out 14)Madras , temperature at noon varies according to -t^2/2 + 8t + 3, where t is elapsed time. Find how much temperature more or less in 4pm to 9pm.? Sol): In equation first put t=9, we will get 34.5........................... (1) Now put t=4, we will get 27.............................. (2) So ans=34.5-27

=7.5 15)Number of edges,faces, vertices and of a cube 1.12,6,8 2.6,8,12 3.4 ,6, 8 4.8,12,6 sol)1.12,6,8 16)What is d largest prime number that can be stored in a 9 bit register? Sol): 2^9=512 509 17) In the word ORGANISATIONAL, if the first & second, third & fourth, forth& fifth are interchanged up to the last letter, what would be the the tenth letter 4m right? Sol): I 18)If VXUPLVH is written as SURMISE, what is SHDVD is written as? Sol): PEASA 19)A power unit is there by the bank of the river of 900 mtr width. a cable is made from power unit to power a plant opposite to that of the river and 3000 mtr away from the power unit. The cost of the cable bellows water Rs5 / mtr and cost of cable on the bank is Rs 4/ mtr. Find the pt where the cable cut through the river. sol)3000-900=2100Ans 20) A, B and C are the mechanisms used separately to reduce the wastage of fuel by 30%, 40% and 10%. What will be the fuel economy if they were used combined? Sol): Method is (70/100*60/100*90/100)*100 = 37.8 So, economy = 100-37.8 = 62.2 21)g[0]=1,g[1]=-1,g[n]=2*g[n-1]-3*g[n-2] then calculate g[4]= ? Sol): try it out 22) Low temperature at the night in a city is 1/3 more than 1/2 high as higher temperature in a day. Sum of the low temperature and highest temp. is 100 degrees. Then what is the low temp? sol)Let highest temp be x so low temp=1/3 of x of 1/2 of x plus x/2 i.e. x/6+x/2 total temp=x+x/6+x/2=100 therefore, x=60 Lowest temp is 40 23)If log 0.317=0.3332 and log 0.318=0.3364 then find log 0.319 ? Sol) log 0.317=0.3332 and log 0.318=0.3364, then log 0.319=log0.318+(log(0.318-0.317)) = 0.3396 24)Match the following 1. Mammal - cow ---> a. A type of 2. Snake - reptile ---> b. A part of 3. Roof - Building ---> c. Not a type of 4. Mushroom - Vegetables ---> d. A superset of Sol): 1-c, 2-d, 3-b, 4-a 25)Find d value of @@+25 - ++@16, where @ denotes square and +

denotes square root? Sol): 621 26)Convert the decimal number 310 to d base 6? Sol): 1234 27) How many positive integer solutions does the equation 2x+3y = 100 have? sol) There is a simple way to answer this kind of Q's given 2x+3y=100, take l. of 'x' coeff and 'y' coeff i.e. l. of 2,3.2*3=6then divide 100 with 6 , which turns out 16 hence answer is 16 28)Amal bought 5 pens, 7 pencils and 4 erasers. Rajan bought 6 pens, 8 erasers and 14 pencils for an amount which was half more than what Amal had paid. What % of the total amount paid by Amal was paid for pens? sol) Let, 5 pens + 7 pencils + 4 erasers = x rupees so 10 pens + 14 pencils + 8 erasers = 2*x rupees also mentioned, 6 pens + 14 pencils + 8 erarsers = 1.5*x rupees so (10-6) = 4 pens = (2-1.5)x rupees so 4 pens = 0.5x rupees => 8 pens = x rupees so 5 pens = 5x/8 rupees = 5/8 of total (note x rupees is total amt paid by amal) i.e 5/8 = 500/8% = 62.5% 29)A work is done by 2 people in 24 min. One of them can do this work alone in 40 min. How much time required to do the same work for the second person? sol)Two people work together in 24 mins. So, their one day work is (1/x)+(1+y)=(1/24) One man can complete the work in 40mins one man's one day work (1/y)= (1/40) Now, (1/x)=(1/24)-(1/40) (1/x)=(1/60) So, x can complete the work in 60 mins 30)A man has to get air-mail. He starts to go to airport on his motorbike. Plane comes early and the mail is sent by a horse-cart. The man meets the cart in the middle after half an hour. He takes the mail and returns back, by doing so, he saves twenty minutes. How early did the plane arrive? sol)10min:::assume he started at 1:00,so at 1:30 he met cart .He returned home at 2:00.so it took him 1 hour for the total jorney.by doing this he saved 20 min.so the actual time if the plane is not late is 1 hour and 20 min.so the actual time of plane is at 1:40.The cart travelled a time of 10 min before it met him.so the plane is 10 min early. 31,32.Two questions were on graphs . Practice graphs of tanx , logx , e^x. 33.34.35. Three questions were on bar graph. It was simple to attempt dere only. No need to practice. 36.37.38. Last problem was based on Venn diagram. Figure was given & on

dat basis 3 ques were dere. Ex: a Venn diagram on no. of people who know only English, only French, only German, both English and French, French and German, German and English, know all the three. ->How many know English more than French? ->What % people speak all the three languages? ->What % people speak German but not English? CRITICAL REASONING SECTION: This is also another scoring section. it consist of three passages with four questions each .Mainly the questions come from 12th edition of Barrons GRE.I prepared for CAT so it is one the easiest section for me. Dont mug from Barrons ...a little bit sense of logical reasoning is enough to score good in this section. TECHNICAL INTERVIEW AND HR ROUND : before going 2 an interview 1.prepare 4 tell me abt ur self ,family background ,strength, weakness type qstn 2.remember all d subs semester wise 3.know about ur project work. they will ask ques on your project work and from fav subjects 4.brush up ur basics in every subject, remember some imp defs n formulae 5.u must knw c language well 6.know abt d company (listen d preplacement talk keenly n write imp points ) 7.give ur answers confidently n wid a smooth voice and you will b through All the Best Guys

1.What is the largest prime number stored in a a) 6 bit pattern (ANS~2^6=64,So no is 61) b) 7 bit pattern (ANS~2^7=128, So no is 127) c) 8 bit pattern (ANS~2^8=256, So no is 251) d) 9 bit pattern (ANS~2^9=512, So no is 503) 2.What is the max 3 digit Prime no? (ANS=997) 3.If G(0)= -1, G(1)=1, G(N)=G(N-1)-G(N-2), then finda) G(2)=? (ANS= 2) b) G(3)=? (ANS= 1) c) G(4)=? (ANS= -1) d) G(5)=? (ANS= -2) e) G(6)=? (ANS= -1) [ HINT~REMEMBER THE QUESTION & THE RIGHT ANS.THIS PROB REQD TIME TO SOLVE.QUESTION IS REPEATED BY SAME DATA MAX TIME. ] 4.If g(1)= -1 & g(0)=1 & g(n)=(3*g(n-1))+(2*g(n-2)) then find g(4)?

5.Which is not a TRIANGLE # a) (2m,3m,4m) b) (3m,4m,7m) c) (3m,5m,9m) [ HINT~SUMMATION OF 2 SIDES > 3RD SIDE. ] # a) (30,60,80) b) (40,60,50) c) (60,30,90) [ HINT~SUMMATION OF 3 ANGLES=180 DEGREE ] 6.Which is the exact power of #Two - Choice : a)2048 b)2068 c)2668 d)2408 [ ANS=2048 ] [ HINT~CHECK THE NO IS DIVISIBLE BY 2^1,2^2,2^3 & SO ON ] # Three - Choice : a)2768 b)2678 c)2187 d)none of these [ ANS=2187 ] # Four - Choice : a)4192 b)2340 c)4096 [ ANS=4096 ] [ HINT~SIMILAR WAY] 7.Complete the series a)3,8,a,24,b,48,63 [ ANS~ a=15, b=35 ] [ HINT~DIFFERENCE IS 5,7,9,11,13,15 ] b)26,19,17,13,11, ,8,7 [ ANS=9] [ HINT~26,17,11,8 DECREASING LIKE 9,6,3 & 19,13,9,7 DECREASING LIKE 6,4,2 ] c)9,10,11,13,15, ,21,28 [ ANS=19 ] [ HINT~9,11,15,21 INCREASING LIKE 2,4,6 & 10,13,19,28 INCRESING LIKE 3,6,9] e) 4, -5, 11, -14, 22, --[ ANS= -27 ] [HINT~] 8.CODING & DECODING a)QJFBTF=PLEASE then HBJO=? [ ANS=GAIN (PREVIOUS LETTER) ] b)TAFJHH=RBEKGI then RBDJK=? [ ANS=PCCKJ (PREVIOUS+ AFFTER LETTER) ] c)TUBUJPO=STATION then FILTER=? [ ANS=SIMILAR TYPE] 9.Interchange 1st & 2nd ,3rd & 4th , and so on, of the words UNIMPRESSIVE & then find 10th letter a)to the left b)from the left [ ANS= a) M, b) S ] [ SIMILAR WORDS=ORGANISATIONAL,SIMULTANEOUSLY ] 10.How many Vs are there under the condition that, S should be followed by V & should not be followed by F.Sequence is=>VSFTWELBVSLLKSMSVFLSDIlike that 11.Select odd one out a)SQL Server,Ingress,Oracle,DB2,Java [ ANS=Java ] bSMPT,ARP,WAP,HTTP,BAAN [ ANS=BAAN ] c)Sybase,Windows NT,Linux,mvs,Java [ ANS=Java ] d)Smalltalk,Eiffel,Lisp,Java [ ANS=Java(NOT SURE) ] e)SQL,DB2,SYBASE,HTTP [ ANS=HTTP ] f)SMPT,WAP,SAP,ARP [ ANS=SAP ]

g)Oracle,Linux,Ingress,DB2 h)Linux,Unix,Solaries,SQL Server

[ ANS=LINUX ] [ ANS=SQL Server ]

12.Find the value of a) @@+25-++@16, where @ denotes square & + denotes square root. [ANS=621] b) $%$6-%$%6, where $ means tripling & % means change of sign. [ANS= -72] c) % # % 6 + # %# 6, % means doubling & # means reciprocal. [ HINT~FIRST CHOOSE GENERAL SYMBOL(LIKE - IN 1ST PROBLEM) & THEN MOVE FROM RIGHT TO LEFT ] 13.a)The no 362 in decimal system is given by 1362 in the X system of numbers.Find X=? [ ANS=6 ] b)In which base of system,decimal no 194 is equal to 1234? OR (194)10=(1234)X Find X=? [ ANS=BASE 5 ] [ HINT= X3*1+X2*2+X1*3+X0*4=194, or X3+2X2+3X=190, or X=5 ]

14.In a two-dimensional array, X (9, 7), with each element occupying 4 bytes of memory, with the address of the first element X (1, 1) is 3000, find the address of X (8, 5). [ ANS= 3212 ] [ HINT~ Formula=Base Add + Byte reqd{N(i-1)+(j-1)} where, Base Add=3000; Byte reqd=4; N=no of coulumns in array=7; i=8; j=5; IN ROW MAJOR ORDER ] 15. 16.A Flight takes off at 2 A.M from northeast direction and travels for 11 hours to reach the destination which is in north west direction.Given the latitude and longitude of source and destination. Find the local time of destination when the flight reaches there? [ ANS: 1:00 P.M ] [ HINT~THIS PROB CAN NOT BE SOLVED AS THE latitude and longitude ARE NOT GIVEN.SO ANS IS LIKE ( 2 A.M. +11 hr) ] 17. My flight takes of at 2am from a place at 18N 10E and landed 10 Hrs later at a place with coordinates 36N70W. What is the local time when my plane landed. a) 6:00 am b) 6:40am c) 7:40 d) 7:00 e) 8:00 (Hint : Every 1 deg longitude is equal to 4 minutes . If west to east add time else subtract time) ANS: (E) 8:00 18. The size of a program is N. And the memory occupied by the program is given by M = square root of 100N. If the size of the program is increased by 1% then how much memory now occupied? 19.A man, a woman, and a child can do a piece of work in 6 days. Man only can do it in 24 days. Woman can do it in 16 days and in how many days child can do the same work? ANS:16 20.Number of faces, vertices and edges of a cube ANS:6,8,12 21. Find the result of the following _expression if, M denotes modulus operation, R denotes round-off, T denotes truncation: M(373,5)+R(3.4)+T(7.7)+R(5.8) ANS:19 22. A power unit is there by the bank of the river of 750 meters width. A cable is made from power unit to power a plant opposite to that of the river and 1500mts away from the power unit. The cost of the cable below water is Rs. 15/- per meter and cost of cable on the bank is Rs.12/-

per meter. Find the total of laying the cable. ANS:20250 23. In Madras , temperature at noon varies according to -t^2/2 + 8t + 3, where t is elapsed time. Find how much temperature more or less in 4pm to 9pm. ANS: 385.8(DB) 24.The size of the bucket is N kb. The bucket fills at the rate of 0.1 kb per millisecond. A programmer sends a program to receiver. There it waits for 10 milliseconds. And response will be back to programmer in 20 milliseconds. How much time the program takes to get a response back to the programmer, after it is sent? ANS: 30MILISECOND 25. A man, a woman, and a child can do a piece of work in 6 days. Man only can do it in 24 days. Woman can do it in 16 days and in how many days child can do the same work? 26. If A, B and C are the mechanisms used separately to reduce the wastage of fuel by 30%, 20% and 10%. What will be the fuel economy if they were used combined. ANS: 20% 27. Which of the following are orthogonal pairs? a. 3i+2j b. i+j c. 2i-3j d. -7i+j ANS: (A)& (C). 28. A can copy 50 papers in 10 hours while both A & B can copy 70 papers in 10 hours. Then for how many hours required for B to copy 26 papers? ANS: 13 29.A is twice efficient than B. A and B can both work together to complete a work in 7 days. Then find in how many days A alone can complete the work? ANS: 10.5 DAYS(11) 30.A finish the work in 10 days. B is 60% efficient than A. So hoW days does B take to finish the work?ANS : 4DAYS. 31.A finishes the work in 10 days & B in 8 days individually. If A works for only 6 days then how many days should B work to complete A?s work? ANS : 3.2 DAYS(4) 32. Find the singularity matrix from a given set of matrices?(Hint det(A)=0) 33.Sum of slopes of 2 perpendicular st. lines is given. Find the pair of lines from the given set of options which satisfy the above condition? Section 3.Critical Reasoning. 1.The players G,H,J,K,L,M,N,O are to be felicitated of representing the county team in Baseball Out of these H,M,O also are in the Football team and K,N are there in the Basket ball team . These players are to be seated on a table and no two players who has represented the county in more than one game are to sit together. 1.Which one of the orders can they be seated in 2. Which of the orders is not possible 3. If N is seated in the middle then which of the following pairs cannot be seated near him . 4. If M is seated then which of the following pairs can be seated next to him. Choices are given for all the questions 2.There are 2 groups named Brown and red. They can?t marry in the same group. If the husband or wife dies then the person will convert to their own group. If a person is married then the husband will have to change his group to his wife?s group. The child will own the mothers group. From these a set of 4 questions were given .Solve them Eg; 1.Brown?s daughter is red (False)

2. If a person is red. Then his/her mother?s brother belong to which group if he is married (Brown) 3.7 people - a,b,c,d,e,f,g Need to make a seating arrangement for them. Conditions: 1)A should be at the center 2) B,F should be at the right extreme 3)C,E always in pair 4)D,G as far as possible Questions from the above were asked? Eg: Which of the following pairs were not possible? Others questions were similar to the above. More questions were from Barrons

You might also like